09.07.2015 Views

moji omiljeni zadaci - Univerzitet u Novom Sadu

moji omiljeni zadaci - Univerzitet u Novom Sadu

moji omiljeni zadaci - Univerzitet u Novom Sadu

SHOW MORE
SHOW LESS

Create successful ePaper yourself

Turn your PDF publications into a flip-book with our unique Google optimized e-Paper software.

PredgovorNa dan kada pišem ovaj predgovor, navršava se tačno 10 godina od smrti PalaErdeša (Erdős Pál, Budapest, 26.3.1913. – Warszawa, 20.9.1996.), velikog majstorateorije brojeva, kombinatorike, i još mnogih drugih oblasti matematike. Kaošto to i naslovna korica sugeriše, ova knjižica posvećena je uspomeni na ovogvelikana. Iz istog razloga, ona sadrži baš 1996 − 1913 = 83 problema. Svojimvišedecenijskim radom u službi kraljice nauka, Erdeš je ostavio nezaobilazan trag,i predstavlja jednu od najneobičnijih, a ujedno najznačajnijih figura matematikeXX veka.Zbirka je, kao što naslov kazuje, sastavljena isključivo od elementarnih zadataka.To znači da za njihovo rešavanje nisu potrebne metode matematičke analize,i da se kao pretpostavljeno predznanje uzimaju samo osnovni, klasični rezultatiaritmetike i veštine obuhvaćene srednjoškolskim gradivom iz algebre. (Kratakpregled najznačajnijih tvr - denja iz teorije brojeva na koja ćemo se u rešenjima eksplicitnopozivati dat je nakon liste zadataka, a pre rešenja.) Uostalom, svi ovdeanalizirani <strong>zadaci</strong> i vode poreklo sa matematičkih takmičenja srednjoškolaca širomsveta u periodu od 1969. do 1995. godine. Izbor je zaista internacionalan (kao štoje i Erdeš bio istinski svetski putnik, gra - danin planete), a bliži podaci o poreklu zadatakadati su uz njihova rešenja. Pri tome, akronim MMO označava Me - dunarodnumatematičku olimpijadu, smotru mladih matematičara koja se organizuje svake godinepočev od 1959. Jedini izuzetak je bila 1980. godina, kada je umesto MMO organizovanonekoliko manjih me - dunarodnih matematičkih turnira (MMT). Najzad,skraćenica BMO označava Balkanske matematičke olimpijade, čija je organizacijapočela 1984.Po kom kriterijumu sam birao zadatke? Teško je odgovoriti na to pitanje, poštobi odgovor na njega bio prilično blizak odre - denju (inače potpuno fluidnog) pojmamatematičke lepote. Negde je to lucidnost i elegancija rešenja, lukava igra detekcijekoja vodi željenom cilju. U drugom slučaju, to je estetika jednostavnosti formulacije,prirodnost problema koja privlači pažnju. U trećem, diskretno nagoveštenamogućnost generalizacije ili suptilnih veza sa ”višom” matematikom (šta god tajtermin označavao), pre svega sa apstraknom algebrom, analitičkom teorijom bro-5


jeva, kombinatorikom. Uglavnom, kriterijum je bio dakako subjektivan, i oslanjaose više na osećaj i iskustvo (mnoge od narednih zadataka sam i sam rešavaopripremajući se svojevremeno za matematička takmičenja), nego na neke racionalneargumente.Ključna primedba koju treba da pomenem u ovom predgovoru jeste da ovo nijeknjiga posvećena ispisivanju rešenja odre - denog skupa zadataka na najkraći mogućinačin. Naprotiv. Ono što mi se čini kao suština ove zbirke jeste pokušaj da se u najvećembroju slučajeva da metodološka analiza postupka rešavanja datih zadataka.Drugim rečima, pokušao sam (kad god je to bilo moguće) da što verodostojnijeprikažem analitički, deduktivni način razmišljanja u procesu otkrivanja rešenja, dapružim kakav-takav uvid u realne misaone tokove i motivišem i opravdam uvo - denjenovih ideja u tom procesu. Naravno da su gotova, ”sintetička” rešenja mnogo konciznija,ponekad i efektnija. Me - dutim, moj prvi cilj je bio da knjiga bude ”userfriendly”,a zakonitosti metodike matematike nalažu sasvim druge prioritete odkonciznosti i efektnosti.Naime, želeo sam da pokažem da se do rešenja matematičkog problema nedolazi gledanjem u kristalnu kuglu (u kojoj iznenada primećujemo egzotične inebulozne matematičke relacije), već da ono nastaje kao rezultat razmišljanja ilogičkog rezonovanja, asocijativnih veza koje se bude i uspostavljaju posmatranjemsastavnih elemenata problema. Matematika je posmatračka disciplina, rekaoje Gaus (a možemo dodati i: detektivska disciplina), pa je suština rešavanja problemau njihovom ”seciranju”, razlaganju na osnovne motive i ideje. Rešenje zadatkamora da, pre svega, zvuči prirodno, tako da deluje potpuno razumno i opravdanoda nam u odre - denom trenutku padne na pamet odre - dena nova ideja. Sjajnoje kada čitalac, proučivši rešenje, zaključi: ”I ja sam mogao da se setim ovoga”.To ne samo da podstiče samopouzdanje, neophodno svakom matematičaru, već iotkriva pravu prirodu matematičkog stila mišljenja. Neki od najboljih primera kojiilustruju ovakav pristup jesu rešenja zadataka br. 5, 8, 14, 17, 51 i 77.Neka od rešenja snabdevena su i dodatnim komentarima. Osim onih tehničkogkaraktera, ti komentari su uglavnom vezani za moguće generalizacije i produbljenjadatih problema, ili pak bliže objašnjavaju odre - dene ideje i detalje.Ova knjižica namenjena je kako - dacima srednjoškolskog uzrasta koji se pripremajuza matematička takmičenja, tako i svima onima koji vole matematiku i uživajuu njoj, svim njenim ljubiteljima. Tako - de, ona može biti korisna i svim studentimakoji izučavaju teoriju brojeva u okviru studija matematike. Na fakultetu na komepredajem (PMF u <strong>Novom</strong> <strong>Sadu</strong>), to su studenti teorijske i primenjene matematikekoji počev od II godine mogu slušati predmet Teorija brojeva (na teorijskojmatematici je on čak i obavezan). Tako - de, to su i studenti master programa smeraprofesor matematike koji izučavaju teoriju brojeva u okviru predmeta Elementarnamatematika I. S obzirom na tako heterogen sastav potencijalnih čitalaca, nije na6


odmet napomenuti da ćemo u ovoj zbirci — za razliku od prakse u matematičkojlogici — pod skupom prirodnih brojeva podrazumevati N = {1, 2, 3, . . .}.Na kraju, želim da izrazim ogromnu zahvalnost koju dugujem svom velikomprijatelju, Žoltu Gajdošu (Gajdos Zsolt), čiji se doprinos u oblikovanju ove zbirkepraktično može meriti sa koautorstvom. Me - dutim, on je moju ponudu da se i njegovoime na - de na koricama džentlmenski odbio, sa lakonskim i savršeno logičnimobrazloženjem: ”Ipak su to tvoji <strong>omiljeni</strong> <strong>zadaci</strong>.” Kako god bilo, veliko mu hvala,sa bitnom napomenom da sve kritike na račun knjige pripadaju samo meni. Tako - de,zahvaljujem se i recenzentima: prof. dr Zoranu Kadelburgu i prof. dr Siniši Crvenkoviću,na izuzetno korisnim sugestijama i pruženoj podršci.NOVI SAD, 20.9.2006.7


Zadaci1. Za koje prirodne brojeve n je broj3 2n+1 − 2 2n+1 − 6 nsložen?2. (a) Pokazati da postoji n ∈ N tako da2 1990 | (1989 n − 1).Naći najmanje takvo n.(b) Neka je m 3 neparan prirodan broj. Odrediti najmanje n za koje2 1989 | (m n − 1).3. Naći sve trojke prirodnih brojeva (a, b, c) takve da proizvod svaka dva broja dajeostatak 1 pri deljenju sa trećim.4. Naći sve prirodne brojeve a, b, c, 1 < a < b < c, takve da(a − 1)(b − 1)(c − 1) | (abc − 1).5. Neka su a, b, c prirodni brojevi, (a, b) = (b, c) = (c, a) = 1. Naći najveći ceobroj koji se ne može predstaviti u obliku xbc + yca + zab, gde su x, y, z 0 celibrojevi.6. Neka ogrlica A ima 14 bisera, a ogrlica B 19 bisera. Dokazati da za svakineparan prirodan broj n postoji način da numerišemo sve bisere brojevima iz skupa{n, n + 1, . . . , n + 32},tako da je svaki broj korišćen tačno jednom, i da su brojevi koji odgovaraju susednimbiserima uzajamno prosti.9


17. Označimo sa S(n) zbir cifara prirodnog broja n u dekadnom zapisu. Broj mzovemo lošim ako se ne može predstaviti u oblikum = n + S(n).Koliko ima loših brojeva — konačno ili beskonačno mnogo?18. Neka je S(n) zbir cifara u dekadnom zapisu prirodnog broja n. Naći sveprirodne brojeve M takve da važiza svako prirodno k M.S(kM) = S(M)19. Neka je f 1 (x) kvadrat zbira cifara prirodnog broja x, aIzračunati f 1991 (2 1990 ).f n (x) = f 1 (f n−1 (x)).20. Neka su a, b, c, d, a < b < c < d, neparni prirodni brojevi za koje važi:(1) ad = bc,(2) a + d = 2 k , b + c = 2 m , za neke prirodne brojeve k, m.Dokazati da je a = 1.21. Broj 9 se može predstaviti kao zbir dva uzastopna prirodna broja 9 = 4 + 5;štaviše, on se može zapisati kao suma bar dva uzastopna prirodna broja na tačnodva načina:9 = 4 + 5 = 2 + 3 + 4.Da li postoji prirodan broj koji se može predstaviti kao zbir 1990 uzastopnih prirodnihbrojeva i koji se može zapisati kao zbir bar dva uzastopna prirodna broja natačno 1990 načina?22. (a) Za koje prirodne brojeve n 3 postoji skup od n uzastopnih prirodnih brojevatakav da je najveći element tog skupa delitelj najmanjeg zajedničkog sadržaocapreostalih brojeva?(b) Za koje prirodne brojeve n 3 postoji jedinstven skup sa gornjim svojstvom?11


23. Neka su a 1 , . . . , a k , b 1 , . . . , b k prirodni brojevi takvi da je (a i , b i ) = 1 za svei, 1 i k. Neka je m najmanji zajednički sadržalac brojeva b 1 , . . . , b k , ac i = a imb i, 1 i k. Dokazati:(a 1 , . . . , a k ) = (c 1 , . . . , c k ).24. Neka je p prost broj, a n prirodan broj. Dokazati da postoji najviše jedanpozitivan broj d takav da važi d | pn 2 i da je n 2 + d potpun kvadrat.25. Neka je k dati prirodan broj. Dokazati da postoji beskonačno mnogo potpunihkvadrata oblika 2 k n − 7.26. Dokazati da postoji prirodan broj k takav da je broj 2 n k+1 složen za sve n ∈ N.27. Neka su m i n prirodni brojevi sa osobinom da za sve prirodne brojeve k važi(11k − 1, m) = (11k − 1, n). Dokazati da tada za neki ceo broj s važi m n = 11s .28. Naći sve aritmetičke progresije u kojima su za sve n ∈ N zbirovi prvih nčlanova potpuni kvadrati.29. Neka je n > 6 i neka su a 1 < a 2 < . . . < a k svi prirodni brojevi manji od n iuzajamno prosti sa n. Dokazati: ako je niz a i aritmetička progresija, tada je n prostbroj ili stepen dvojke.30. Neka je n ∈ N. Dokazati da postoje različiti brojevi a, b, c ∈ N tako da jei a | bc.n 2 < a, b, c < n 2 + n + 3 √ n31. Za prirodne brojeve a, b, c, d važi ad = bc i a < b < c < d. Dokazati da postojiprirodan broj n tako da je a < n 2 < d.32. Naći sve prirodne brojeve n < 1978 sa osobinom: ako je prirodan broj m,1 < m < n, uzajamno prost sa n, tada je on prost.33. Predstaviti broj 5 1985 − 1 kao proizvod tri prirodna broja od kojih je svaki većiod 5 100 .34. Neka je p(x) polinom sa celim koeficijentima takav da je p(0) = p(1) = 1.Neka je a 1 proizvoljan ceo broj. Definišemo niz a 1 , a 2 , . . . , a n , . . . tako da za sven 1 važia n+1 = p(a n ).Dokazati da je svaki par različitih elemenata ovog niza uzajamno prost.12


35. Neka je n > 1. Dokazati da sledeći polinom nema racionalne nule:p n (x) =n∑k=0x kk! .36. Neka je f(x) =⌊√x 2 +⌋x + p, gde je p ∈ N. Dokazati: ako je f(x) prost broj zasve x ∈ {0, 1, . . . , p3}, tada je f(x) prost broj za sve x ∈ {0, 1, . . . , p − 2}.37. Dat je polinomf(x) = x 8 + 4x 6 + 2x 4 + 28x 2 + 1.Neka je p > 3 prost broj takav da postoji z ∈ N za koji p | f(z). Dokazati dapostoje celi brojevi z 1 , . . . , z 8 takvi da za g(x) = (x − z 1 ) . . . (x − z 8 ) važi da susvi koeficijenti polinoma f(x) − g(x) deljivi sa p.38. Neka je k 2 proizvoljan prirodan broj. Dokazati da postoji iracionalan brojr k takav da za svaki prirodan broj n važi:⌊rk n ⌋ ≡ −1(mod k).39. Dokazati: ako prirodan broj A nije potpun kvadrat, tada postoji prirodan broj ntako da je A = ⌊ n + √ n + 1 2⌋.40. Dokazati da niz brojeva ⌊n √ 2⌋, n ∈ N, sadrži beskonačno mnogo potpunihkvadrata.41. Dokazati da niz brojeva ⌊n √ 2⌋, n ∈ N, sadrži beskonačno mnogo stepenadvojke.42. Neka je n ceo broj. Dokazati: ako je brojprirodan, onda je on potpun kvadrat.2 + 2 √ 28n 2 + 143. Za koje prirodne brojeve n postoji prirodan broj m tako da ni jedan od brojevam + 1, m + 2, . . . , m + n nije stepen prostog broja?44. Naći najveći prirodan broj k za koji1991 k ∣ ∣∣(1990 19911992 + 1992 19911990) .13


53. Dokazati da za sve prirodne brojeve m postoji prirodan broj n > m takav da sedekadni zapis broja 5 n dobija dopisivanjem izvesnog broja cifara sleva dekadnomzapisu broja 5 m .54. Dokazati da postoji beskonačno mnogo prirodnih brojeva n takvih da se udekadnom zapisu broja 5 n pojavljuje 1976 uzastopnih nula.55. Dokazati da1989∣(n nnn − n nn)za sve prirodne brojeve n 3.56. (a) Dokazati da postoji beskonačno mnogo parova prironih brojeva (m, n)takvih da je 4mn − m − n + 1 potpun kvadrat.(b) Dokazati da ne postoji nijedan par prirodnih brojeva (m, n) takav da je 4mn −m − n potpun kvadrat.57. (a) Dokazati da za svaki prirodan broj a 3 postoji beskonačno mnogo prirodnihbrojeva n za koje n | (a n − 1).(b) Naći sve prirodne brojeve n za koje n | (2 n − 1).(c) Neka je k 2 i neka su n 1 , n 2 , . . . , n k prirodni brojevi takvi dan i+1 |(2 n i− 1)za sve 1 i k − 1, kao i n 1 |(2 n k− 1). Dokazati da je n 1 = . . . = n k = 1.58. Dokazati da je za svaki prirodan broj n ∈ N, nizkonstantan počev od nekog člana.2, 2 2 , 2 22 , 2 222 , . . . (mod n)59. Naći sve prirodne brojeve n za koje n 2 | (2 n + 1).60. Naći sve prirodne brojeve n za koje jed 2 1 + d 2 2 + d 2 3 + d 2 4 = n,gde su 1 = d 1 < d 2 < . . . < d k = n (k 4) svi pozitivni delioci broja n.61. Rešiti u skupu celih brojeva:x 5 − x 3 − x 2 + 1 = y 2 .15


62. Da li jednačinax 2 + xy + y 2 = 2ima racionalna rešenja?63. Naći sva celobrojna rešenja jednačinea 2 + b 2 + c 2 = a 2 b 2 .64. Naći sva celobrojna rešenja jednačinex 3 − y 3 = 2xy + 8.65. Naći sva celobrojna rešenja jednačinex 3 + x 2 y + xy 2 + y 3 = 8(x 2 + xy + y 2 + 1).66. Dokazati da jednačinax 2 + 5 = y 3nema rešenja u skupu celih brojeva.67. Neka je n 2 prirodan broj. Dokazati da jednačinax n + 1 = y n+1nema rešenje x, y ∈ N za koje važi (x, n + 1) ≠ 1.68. Naći sve prirodne brojeve x, y za koje važi:7 x − 3 · 2 y = 1.69. Naći sve prirodne brojeve x, y, z za koje važi:3 x + 4 y = 5 z .70. (a) Prirodni brojevi x, y su takvi da je brojceo i deli 1978. Dokazati da je x = y.x 2 + y 2x + y(b) Dokazati da na kružnici opisanoj oko kvadrata sa temenima (0, 0), (1978, 0),(1978, 1978), (0, 1978), nema celobrojnih tačaka, sem navedenih.16


71. (a) Neka za prirodne brojeve x, y, z važi xy − z 2 = 1. Dokazati da postojenenegativni celi brojevi a, b, c, d tako da jex = a 2 + b 2 , y = c 2 + d 2 , z = ac + bd.(b) Dokazati: ako je p prost broj i p ≡ 1(mod 4), tada se p može predstaviti kaozbir dva kvadrata prirodnih brojeva.72. Neka su a, b prirodni brojevi, i neka se pri deljenju a 2 + b 2 sa a + b dobijakoličnik q i ostatak r. Naći sve parove (a, b) za koje je q 2 + r = 1977.73. Dokazati: ako su a i b prirodni brojevi i a 2 + b 2 − a je deljivo sa 2ab, tada je apotpun kvadrat.74. Naći sva rešenja jednačineu skupu prirodnih brojeva.x 2 − 5xy + y 2 + 5 = 075. Dokazati: ako je za neke prirodne brojeve a, b brojceo, tada je on potpun kvadrat.76. (a) Neka je n ∈ N. Ako jednačinaa 2 + b 2ab + 1x 3 − 3xy 2 + y 3 = nima rešenje u skupu prirodnih brojeva, tada ona ima bar tri različita rešenja u skupuprirodnih brojeva. Dokazati.(b) Dokazati da gornja jednačina za n = 2891 nema rešenja u skupu prirodnihbrojeva.77. Neka su a, b, c celi brojevi različiti od 0. Poznato je da jednačinaax 2 + by 2 + cz 2 = 0ima celobrojno rešenje različito od x = y = z = 0. Dokazati da jednačinaima racionalno rešenje.ax 2 + by 2 + cz 2 = 117


78. Neka su a, b celi brojevi koji nisu potpuni kvadrati. Dokazati: ako jednačinax 2 − ay 2 − bz 2 + abw 2 = 0ima netrivijalno celobrojno rešenje, tada to važi i za jednačinu79. Naći celobrojno rešenje jednačinex 2 − ay 2 − bz 2 = 0.x 2 1 + x 2 2 + . . . + x 2 29 = 29x 1 x 2 . . . x 29tako da za bar jedno 1 k 29 važi x k 1988 2 .80. Rešiti jednačinu u skupu prirodnih brojeva:x 2n+1 − y 2n+1 = xyz + 2 2n+1 ,pri čemu važe ograničenja n 2 i z 5 · 2 2n .81. Naći sve prirodne brojeve x, y za koje važigde je z = (x, y).x + y 2 + z 3 = xyz,82. Neka je n prirodan broj, A skup koji se sastoji od tačno n+1 prirodnih brojeva,a P skup svih prostih faktora elemenata skupa A. Ako je |P | n, dokazati dapostoji B ⊆ A, B ≠ ∅, tako da je proizvod elemenata skupa B potpun kvadrat.83. Konstruisati funkciju f : Q + → Q + (gde Q + označava skup pozitivnih racionalnihbrojeva) tako da važiza sve x, y ∈ Q + .f(xf(y)) = f(x)y18


Nekoliko poznatih teoremaEuklidov stavNeka je x 1 , . . . , x n , y ∈ Z i n 2. Postoje celi brojevi α 1 , . . . , α n takvi da jeα 1 x 1 + . . . + α n x n = yako i samo ako (x 1 , . . . , x n ) | y. Specijalno, brojevi x 1 , . . . , x n su uzajamno prosti(tj. (x 1 , . . . , x n ) = 1) ako i samo ako važiza neke α 1 , . . . , α n ∈ Z.Kineska teorema o ostacimaα 1 x 1 + . . . + α n x n = 1Neka su m 1 , . . . , m n prirodni brojevi takvi da je (m i , m j ) = 1 za sve 1 i ≠j n, i neka su r 1 , . . . , r n proizvoljni celi brojevi. Tada sistem kongruencijax ≡ r 1 (mod m 1 ),.x ≡ r n (mod m n ),ima rešenje. Štaviše, to rešenje je jedinstveno u skupu{0, 1, . . . , M − 1},gde je M = m 1 . . . m n . Ako sa x 0 označimo to rešenje, tada su sva preostalarešenja oblika x = x 0 + kM, k ∈ Z.Osnovna teorema aritmetikeSvaki prirodan broj n 2 se može zapisati u oblikun = p α 11 . . . pα kk ,19


gde su α 1 , . . . , α k 1 prirodni brojevi, a p 1 , . . . , p k različiti prosti brojevi. Pritome je gornji prikaz jedinstven do na permutaciju prostih faktora.Bertranov stavZa svaki prirodan broj n 2 postoji prost broj p takav da je n < p < 2n.Stoga, ako su p i i p i+1 dva uzastopna prosta broja, važiLežandrova teoremap i+1 < 2p i .Najviši stepen kojim prost broj p deli n! je jednak⌊ ⌋ ⌊ ⌋ ⌊ ⌋n n n+p p 2 +p 3 + . . .Ojlerova i mala Fermaova teoremaNeka je n 2 prirodan broj i neka ϕ(n) označava broj prirodnih brojeva n iuzajamno prostih sa n (Ojlerova funkcija). Tada za sve a ∈ N takve da (a, n) = 1važia ϕ(n) ≡ 1(mod n).Specijalno, ako je n prost broj, tada je ϕ(n) = n − 1, pa važi: ako n ∤ a, tada jeVilsonova teoremaNeka je p prost broj. Tada jea n−1 ≡ 1(mod n).(p − 1)! ≡ −1(mod p).20


Rešenja1. S.S.S.R., 1990.Za koje prirodne brojeve n je brojsložen?3 2n+1 − 2 2n+1 − 6 nRešenje. Dati izraz možemo zapisati u obliku3 · (3 n ) 2 − 2 · (2 n ) 2 − 3 n · 2 n .Nakon smene x = 3 n , y = 2 n , dobijamo 3x 2 − 2y 2 − xy, što se može rastaviti nasledeći način:3x 2 − 2y 2 − xy = 3x 2 − 3xy + 2xy − 2y 2 = (x − y)(3x + 2y).Uvrštavajući vrednosti za x, y, imamo3 2n+1 − 2 2n+1 − 6 n = (3 n − 2 n )(3 n+1 + 2 n+1 ).Kako za n 2 važi 3 n − 2 n > 1, to je dati broj složen za sve prirodne brojeven ≠ 1. Za n = 1, on je jednak prostom broju 13.Komentar. Potpuno analognim postupkom, moguće je pokazati da je za a 2 in 2 broj (a + 1) 2n+1 − a 2n+1 − (a(a + 1)) n složen.21


2. (b): Predlog za MMO, 1989. (Rumunija); (a): Ma - darska, 1990.(a) Pokazati da postoji n ∈ N tako daNaći najmanje takvo n.2 1990 | (1989 n − 1).(b) Neka je m 3 neparan prirodan broj. Odrediti najmanje n za koje2 1989 | (m n − 1).Rešenje. Rešićemo zadatak koji je opštiji od (b): odredićemo minimalno n za koje2 k | (m n − 1), gde je k dati prirodan broj.Napišimo n = 2 t q, gde je q neparan broj. Imamo faktorizaciju[]m n − 1 = (m 2t ) q − 1 = (m 2t − 1) (m 2t ) q−1 + . . . + m 2t + 1 .Broj u uglastoj zagradi je neparan (pošto je reč o zbiru q neparnih brojeva), pa2 k | (m n − 1) ako i samo ako 2 k | (m 2t − 1). Otuda sledi da je q = 1 za traženominimalno n.S druge strane, važim 2t − 1 = (m 2 − 1)(m 2 + 1) . . . (m 2t−1 + 1).Budući da je ovde m neparan broj, m 2 daje ostatak 1 pri deljenju sa 4, a isto važii za broj oblika m 2r , r 1. Zbog toga su u gornjem proizvodu sa desne stranesvi činioci sem prvog deljivi sa 2, ali ne i sa 4, što znači da je najviši stepen kojimdvojka deli (m 2 + 1) . . . (m 2t−1 + 1) jednak t − 1. Prema tome, prostaje da serazmotri stepen dvojke u faktoru m 2 − 1 = (m − 1)(m + 1).Kako m može davati ostatak 1 ili 3 (mod 4), posebno razmatramo ova dvaslučaja. Ako je m ≡ 1(mod 4), uočimo najveći broj s 2 sa osobinom da2 s | (m − 1). Tada je m + 1 deljiv sa 2, ali ne i sa 4, pa je najviši stepen kojim2 deli m 2 − 1 jednak s + 1. S druge strane, ako je m ≡ 3(mod 4), posmatramonajveći broj s 2 za koji važi 2 s | (m + 1). Slično kao i malopre, sledi da je brojm 2 − 1 deljiv sa 2 s+1 , ali ne i sa 2 s+2 .Dakle, ako je broj s odreden - kao što je to opisano u prethodnom pasusu (aon zavisi isključivo od m), tada je najviši stepen kojim 2 deli m 2t − 1 jednak(t − 1) + (s + 1) = t + s. Stoga je traženo minimalno rešenje n = 2 k−s u slučajus k, u suprotnom je u pitanju n = 1.U zadatku (a) je m = 1989 = 4 · 494 + 3, pa je u tom slučaju s = 2, dokje k = 1990, što znači da je rešenje zadatka n = 2 1988 . Za zadatak pod (b) trebaprimeniti gornje rešenje za k = 1989.22


3. SR Nemačka, 1990.Naći sve trojke prirodnih brojeva (a, b, c) takve da proizvod svaka dva broja dajeostatak 1 pri deljenju sa trećim.Rešenje. Uslov zadatka možemo formulisati na sledeći način: postoje celi brojeviα, β, γ tako da jeab − 1 = γc, bc − 1 = αa, ca − 1 = βb,za posmatrane prirodne brojeve a, b, c. Primetimo da pri tome a, b, c moraju bitirazličiti od 1. Množenjem gornjih jednakosti, dobijamo:αβγabc = (abc) 2 − abc(a + b + c) + ab + bc + ca − 1.Prebacujući na jednu stranu sve članove koji sadrže abc, zaključujemo da postojiprirodan broj µ tako da jeµabc = ab + bc + ca − 1. (1)Dalje, razlikujemo dva slučaja: ako su neka dva broja jednaka, npr. a = b, tada važiαa = ac − 1. Sledi a = 1, što je nemoguće. Znači, moguć je samo drugi slučaj,kada su brojevi a, b, c različiti. Neka je, na primer, a < b < c. Tada imamo, naosnovu gornje jednakosti i upravo usvojenog poretka:abc < ab + bc + ca < 3bc,pa je a < 3, tj. a = 2. Uvrštavajući u (1), imamo:pa zaključujemo:(2µ − 1)bc = 2(b + c) − 1,bc < 2(b + c) < 4c,tj. b < 4. Kako je b > a = 2, mora biti b = 3. Otuda slediγc = 5,pa je c = 5. Sada se sva rešenja zadatka dobijaju kao permutacije trojke (2, 3, 5).23


4. MMO, 1992. (Novi Zeland)Naći sve prirodne brojeve a, b, c, 1 < a < b < c, takve daRešenje. Najpre, uočimo:(a − 1)(b − 1)(c − 1) | (abc − 1).(a − 1)(b − 1)(c − 1) = abc − (ab + bc + ca) + a + b + c − 1 < abc − 1,pa sledi da mora bitiabc − 1 2(a − 1)(b − 1)(c − 1). (2)Iz gornje nejednakosti se nakon množenja i sre - divanja dobija:abc 2(ab + bc + ca) − 2(a + b + c) + 1 < 6bc, (3)pa je a 5. Razmotrićemo posebno svaki od slučajeva a ∈ {2, 3, 4, 5}.Dalju analizu će na ovom mestu znatno ubrzati primedba da a, b, c moraju bitiiste parnosti. Naime, ako je bar jedan od tih brojeva paran, tada je abc − 1 neparanbroj, pa on nema parnih delilaca; zato u tom slučaju sva tri broja a, b, c moraju bitiparna.Ako je a = 2, tada iz (2) sledi2bc − 1 > 2(b − 1)(c − 1)(stroga nejednakost važi zbog različite parnosti leve i desne strane), pa imamo2bc − 1 3(b − 1)(c − 1),odakle je bc 3b + 3c − 4 < 6c, b 5. Po malopre - dašnjoj primedbi, sledi da jeb = 4, pa se ispitivani uslov svodi na3(c − 1) | (8c − 1) = 8(c − 1) + 7.Znači, c − 1 = 7, tj. c = 8. Sada neposredno proveravamo da trojka (2, 4, 8) jestejedno od rešenja zadatka.Pre nego što pre - demo na preostale slučajeve, transformišimo (3) u sledeći oblik:Pošto je b < c, imamo(a − 2)bc (2a − 2)b + (2a − 2)c − (2a − 1).(2a − 2)b + (2a − 2)c − (2a − 1) < (4a − 4)c − (2a − 1) < 4(a − 1)c,24


odakle je (a − 2)bc < 4(a − 1)c. Odavde, pod pretpostavkom a ≠ 2, sledib < 4 a − 1a − 2 . (4)Ovu nejednakost ćemo koristiti u analizi sva tri preostala slučaja.Dakle, ako je a = 3, tada po (4) imamo b < 8, tj. b ∈ {5, 7}. Za b = 5, iz8(c − 1) | (15c − 1) = 15(c − 1) + 14zaključujemo da je c = 15 (podsetimo se, c mora biti neparno), što tako - de zadovoljavauslove zadatka, pa smo dobili rešenje (3, 5, 15). U drugom slučaju, imamob = 7 i12(c − 1) | (21c − 1).Me - dutim, ova relacija je nemoguća, jer −1 nije deljivo sa 3.U slučaju a = 4 nejednakost (4) povlači 4 < b < 6, i odmah imamo kontradikciju,budući da b mora biti parno.Najzad, u slučaju a = 5 iz (4) sledi b 5, što je nemoguće, pošto je pouslovima zadatka b > a = 5.Prema tome, skup rešenja je{(2, 4, 8), (3, 5, 15)}.25


5. MMO, 1983. (SR Nemačka)Neka su a, b, c prirodni brojevi, (a, b) = (b, c) = (c, a) = 1. Naći najveći ceo brojkoji se ne može predstaviti u obliku xbc+yca+zab, gde su x, y, z 0 celi brojevi.Rešenje. Označimo traženi broj sa M. Rešenje započinjemo razmatranjima koja ćenam omogućiti da otkrijemo njegovu vrednost.Kao što je to dato uslovima zadatka, mi želimo da postignemo sledeće: za sveprirodne brojeve n > M, jednačinabcx + cay + abz = nima nenegativno rešenje (x, y, z), dok za n = M to nije slučaj.Jasno, kako je (a, b) = (b, c) = (c, a) = 1, sledi da važi (ab, bc, ca) = 1, pa poEuklidovom stavu za svaki prirodan broj n postoje celi brojevi x 0 , y 0 , z 0 tako da jebcx 0 + cay 0 + abz 0 = n.Prema tome, naš problem se svodi na to da se ustanovi kada postoji nenegativnocelo rešenje jednačinebc(x − x 0 ) + ca(y − y 0 ) + ab(z − z 0 ) = 0. (5)Iz ove jednačine sledi da a | (x−x 0 ) i b | (y−y 0 ). Drugim rečima, važi x = x 0 +asi y = y 0 + bt za neke s, t ∈ Z. S druge strane, za proizvoljne s, t ∈ Z i x, y kojisu navedenog oblika, nakon uvrštavanja u (5) dobijamo da je z = z 0 − c(s + t).Prema tome,{(x 0 + as, y 0 + bt, z 0 − c(s + t)) : s, t ∈ Z} (6)predstavlja skup svih celobrojnih rešenja jednačine (5). Sada je jasno da je pitanjepredstavljivosti broja n u traženom obliku ekvivalentno postojanju trojke iz gornjegskupa koja se sastoji od nenegativnih brojeva.Iz (6) se vidi da vrednost z, treće komponente rešenja, raste ako s, t opadaju.Pošto pokušavamo da u posmatranom skupu na - demo trojku nenegativnih brojeva,uočićemo s 0 , t 0 ∈ Z koji su minimalni sa osobinom da je x 0 +as 0 0 i y 0 +bt 0 0. Označimo, pri tome, x 1 = x 0 + as 0 , y 1 = y 0 + bt 0 i z 1 = z 0 − c(s 0 + t 0 ). Kakotada za proizvoljno nenegativno rešenje (x, y, z) jednačine (5) mora biti x x 1 iy y 1 , sledi da je abz = n−bcx−cay n−bcx 1 −cay 1 = abz 1 , tj. 0 z z 1 ,zaključujemo da je egzistencija nenegativnog rešenja u skupu (6) ekvivalnetna saz 1 0.Pošto je očito 0 x 1 a − 1 i 0 y 1 b − 1, dobijamo da važiabz 1 = n − bcx 1 − cay 1 n − bc(a − 1) − ca(b − 1) =26


= n − (2abc − bc − ca).Kako je željena nejednakost z 1 0 ekvivalentna sa z 1 > −1, tj. sa abz 1 > −ab,zaključujemo da se svaki broj n koji zadovoljava n − (2abc − bc − ca) > −ab,odnosno n > 2abc − ab − bc − ca, može predstaviti na traženi način.Zbog toga, zadatak će biti rešen ukoliko pokažemo da 2abc − ab − bc − ca nijemoguće prikazati u obliku bcx + cay + abz za neke x, y, z 0. U suprotnom,nakon sre - divanja bismo imalibc(x + 1) + ca(y + 1) + ab(z + 1) = 2abc,pa bi zbog a | (x + 1) i x 0 sledilo x + 1 a, a slično i y + 1 b i z + 1 c.Odatle bismo dobili2abc cba + cab + abc = 3abc,što je kontradikcija. Znači, traženi broj je M = 2abc − ab − bc − ca.Komentar 1. Izvorna formulacija zadatka je bila da se dokaže da je 2abc − ab −bc − ca najveći ceo broj koji se ne može prikazati u obliku bcx + cay + abz zaneke cele x, y, z 0. Izmenom formulacije se dobija teži, ali i mnogo zanimljivijizadatak.Komentar 2. Može se pokazati da važi uopštenje tvrdenja - zadatka: naime, ako sua 1 , a 2 , . . . , a n prirodni brojevi takvi da je (a i , a j ) = 1 za sve 1 i ≠ j n, tadaje)n∑ 1a 1 a 2 . . . a n(n − 1 −a inajveći ceo broj koji se ne može prikazati u oblikui=1x 1 a 2 . . . a n + a 1 x 2 . . . a n + . . . + a 1 . . . a n−1 x n ,gde su x 1 , x 2 , . . . , x n nenegativni celi brojevi.27


6. S.A.D., 1990.Neka ogrlica A ima 14 bisera, a ogrlica B 19 bisera. Dokazati da za svaki neparanprirodan broj n postoji način da numerišemo sve bisere brojevima iz skupa{n, n + 1, . . . , n + 32},tako da je svaki broj korišćen tačno jednom, i da su brojevi koji odgovaraju susednimbiserima uzajamno prosti.Rešenje. Osnovna ideja rešenja je da što je moguće više koristimo uzastopneprirodne brojeve za označavanje susednih bisera, pošto su oni uzajamno prosti.Pokušaćemo da iz datog skupa na pogodan način izdvojimo 14 uzastopnih brojevakojima ćemo označiti bisere ogrlice A, naime,n + m, n + m + 1, . . . , n + m + 13(gde će broj m, 1 m 18 biti naknadno odre - den), dok će preostali brojevi činitidva niza uzastopnih brojeva dužine m, odnosno 19 − m. U pitanju sun, n + 1, . . . , n + m − 1, n + m + 14, . . . , n + 32,i njima ćemo, ne menjujući im redosled, označiti bisere ogrlice B. Primetimonajpre da je na ogrlici B ispunjen uslov (n, n + 32) = (n, 32) = 1, jer je n neparanbroj. Prema tome, uslovi iz kojih odre - dujemo m su(n + m, n + m + 13) = 1, (n + m − 1, n + m + 14) = 1.Važe sledeće jednakosti:pa m tražimo iz sledeća tri uslova:(1) m ≢ −n(mod 13),(2) m ≢ 1 − n(mod 3),(3) m ≢ 1 − n(mod 5).(n + m, n + m + 13) = (n + m, 13),(n + m − 1, n + m + 14) = (n + m − 1, 15),Me - dutim, od brojeva 1, 2, . . . , 18 najviše dva ne zadovoljavaju prvi uslov, tačnošest ne zadovoljavaju drugi i najviše četiri treći uslov. Prema tome, me - du njimapostoji broj m 0 koji ispunjava sva tri uslova. Ako sada stavimo m = m 0 , ondanumeracija obe ogrlice ima tražene osobine.28


7. Predlog za MMO, 1987. (Ma - darska)Dokazati: postoje celi brojevi a 1 , . . . , a m i b 1 , . . . , b k takvi da brojevi a i b j , 1 i m, 1 j k, čine potpun sistem ostataka po modulu mk ako i samo ako važi(m, k) = 1.Rešenje. (⇒) Neka brojevi a i b j , 1 i m, 1 j k, čine potpun sistemostataka po modulu mk. Tada je tačno jedan od njih deljiv sa mk; na primer,neka mk | a 1 b 1 . Sledi da postoje brojevi a ′ | a 1 i b ′ | b 1 tako da je mk = a ′ b ′ .Sada ne može biti a ′ | (a i − a s ) za neke indekse i ≠ s, jer bi u suprotnom važilomk = a ′ b ′ | (a i b 1 −a s b 1 ), što je u suprotnosti sa početnom pretpostavkom. Odavdezaključujemo da mora biti a ′ m; u suprotnom bi od m brojeva a i , bar dva davalaisti ostatak po modulu a ′ . Slično, b ′ k, pa zbog mk = a ′ b ′ sledi a ′ = m,b ′ = k, što znači da brojevi a i , b j redom čine potpun sistem ostataka po modulu m,odnosno k.Pretpostavimo sada da je (m, k) > 1 i neka je p prost broj takav da p | (m, k).Tada me - du brojevima a i ima m − m p onih koji nisu deljivi sa p. Analogno, me - dubrojevima b j , k − k p nisu deljivi sa p. Znači, (m − m p )(k − k p ) brojeva a ib j nijedeljivo sa p. Medutim, - kako ti brojevi čine potpun sistem ostataka po modulu mk,to medu - njima ima mk − mkponih koji nisu deljivi sa p. Najzad,(m − m ) (k − k ) (≠ mk − mk )p ppdaje kontradikciju. Dakle, mora biti (m, k) = 1.(⇐) Neka je (m, k) = 1. Naš cilj je da navedemo dva niza celih brojevaa 1 , . . . , a m i b 1 , . . . , b k tako da proizvodi oblika a i b j čine potpun sistem ostatakapo modulu mk.Kao što smo to već pokazali, tada a i , 1 i m, mora činiti potpun sistemostataka po modulu m, dok brojevi b j , 1 j k, moraju davati sve različiteostatke po modulu k. Želimo da ustanovimo u kakvom su odnosu brojevi a i i brojk. Polazeći od pretpostavke da je k > 1, neka je q prost faktor od k. Neka medu-brojevima a i ima tačno x onih koji nisu deljivi sa q. Analogno kao i malopre,medu - brojevima b j ima k − k q onih koji nisu deljivi sa q. S druge strane, me du -proizvodima a i b j ima mk − mk onih koji nisu deljivi sa q. Otuda jexq(k − k )q= mk − mkq ,tj. x = m. Zbog toga, nijedan od brojeva a i ne može biti deljiv sa q. Pošto je prostfaktor q od k bio proizvoljno odabran, sledi da je (a i , k) = 1 za sve 1 i m.Analogno, (b j , m) = 1 za sve 1 j k.29


Prethodna razmatranja sugerišu da uočimo sledeće brojeve:a i = ki + 1, 1 i m,b j = mj + 1, 1 j k.Očigledno, a r − a s = k(r − s), što zbog (m, k) = 1 i |r − s| < m povlači dasu svi a i različiti (mod m). Slično, brojevi b j su svi različiti (mod k). Štaviše,(a i , k) = (b j , m) = 1 za sve i, j. Ukoliko bi brojevi a i b j i a r b s davali isti ostatakpri deljenju sa mk, tada bi bilo:mk | (a i b j − a r b s ) = (ki + 1)(mj + 1) − (kr + 1)(ms + 1) == km(ij − rs) + m(j − s) + k(i − r).Zbog (m, k) = 1, moralo bi biti k | (j − s) i m | (i − r), što znači i = r i j = s.Prema tome, cilj je postignut, tj. navedeni brojevi imaju traženu osobinu.30


8. Predlog za MMO, 1990. (Irska)Ekscentrični matematičar se kreće po lestvama sa n prečki, tako što prelazi odjednoma prečki kada se kreće nagore, a b prečki kada se spušta nadole. Nizom korakanaviše i naniže, on se sa zemlje penje na vrh lestvi i zatim silazi ponovo na zemlju.Naći najmanje n za koje je ovo moguće.Rešenje. Bez ograničenja opštosti, možemo pretpostaviti da je a b: u suprotnom,zamenimo uloge brojeva a i b, pa tada svakom ”uspešnom” nizu koraka u prvobitnojpostavci odgovara, kada taj niz koraka posmatramo unazad, uspešan niz koraka unovoj postavci.Takode, - opšti slučaj se može svesti na slučaj kada je (a, b) = 1. Naime, ako je(a, b) = d > 1, tada je jasno da matematičar može dospeti samo do onih prečki čijisu redni brojevi deljivi sa d (gde zemlju posmatramo kao nultu prečku). Stoga jesvaki niz koraka koji matematičar može preduzeti u očiglednoj bijektivnoj korespondencijisa odgovarajućim nizom koraka u situaciji kada su umesto parametarazadatka a, b dati redom a d i b d. Zbog toga, ako sa n(a, b) označimo traženi broj, tadaje( an(a, b) = nd d), b d.Prema tome, u daljem možemo pretpostaviti da je (a, b) = 1.U načelu, možemo razlikovati dva tipa nizova koraka koje matematičar može dapreduzme. Prva mogućnost je da on u jednom trenutku dvaput uzastopno prelazi poa prečki naviše. Me - dutim, da bi to bilo moguće, potrebno je da imamo na raspolaganju 2a prečki. Mi ćemo pokazati da matematičar može da obavi traženizadatak i za n koje je manje od 2a, a to je moguće samo ukoliko posmatramo drugitip nizova koraka, kod kojih se ne pojavljuju dva uzastona koraka naviše. Drugimrečima, nakon svakog koraka od a prečki naviše sledi nekoliko (bar jedan) korakaod po b prečki nadole. Kako bismo minimizovali n, broj prečki na lestvama, intuitivnoje opravdano da posmatramo (u izvesnom smislu ”ekstreman”) niz koraka,definisan sledećim algoritmom.1. Na početku, matematičar je na nivou 0.2. Ukoliko se matematičar nalazi (posle i-te iteracije ovog algoritma) na nivour i < b (kada ne može da se kreće nadole), tada preduzima jedan korak naviše,do prečke a + r i .3. Sa prečke a+r i , matematičar silazi za po b prečki naniže onoliko puta kolikoje to moguće, dok ne do - de do prečke r i+1 < b.31


4. Ako je r i+1 > 0, vratimo se na korak 2. U suprotnom, algoritam je završen.Posmatrajući korak br. 2, jasno je da je ovaj algoritam moguće sprovesti do krajaukoliko je n a + b − 1.Pokažimo sada da za n = a + b − 1 opisani algoritam upravo proizvodi nizkoraka koji zadovoljava zahteve zadatka. Najpre, induktivnim argumentom se lakopotvr - duje da važi r i ≡ ia (mod b). Zaista, sa r i -te prečke, matematičar najpreprelazi na prečku a + r i , a zatim nekoliko (recimo, q i ) puta silazi po b prečkinadole, sve dok ne do - de na prečku r i+1 = a + r i − q i b < b. Odavde se odmahvidi da r i ≡ ia (mod b) povlači r i+1 ≡ (i + 1)a (mod b). S jedne strane, dobijenakongruencija znači da ćemo nakon b iteracija imati r b ≡ ba ≡ 0 (mod b) (tj. da ćese matematičar vratiti na zemlju). S druge strane, pošto je (a, b) = 1, postoji j < btako da je r j ≡ ja ≡ b−1 (mod b). Stoga u narednom koraku matematičar dospevana vrh lestvi, pa zaključujemo da posmatrani niz koraka ima željene osobine.Preostaje da pokažemo da za n = a + b − c, gde je c 2, matematičar ne možeda realizuje postavljeni zadatak. Zapravo, jedini način na koji on može da se krećena lestvama je upravo onaj opisan u gornjem algoritmu. Naime, na svakom nivouon ima na raspolaganju najviše jedan korak: na nivoima b − c on može samoda ide a prečki nagore, na nivoima b može samo da ide b prečki nadole, a naprečkama b − c + 1, . . . , b − 1 nema na raspolaganju nijedan potez (tj. ”zaglavio”se). Me - dutim, kako ia ≡ 0 (mod b) povlači b | i, on ne može ispuniti zadatak prenego što ne obavi bar b iteracija. Ali, kako je ja ≡ b − 1 (mod b) za neko j < b,on će se sigurno zaglaviti pre toga: ili se zaglavio pre j-te iteracije, ili ako nije,dospeva na prečku b − 1 i zaglavljuje se upravo u j-toj iteraciji.Dakle, ako je (a, b) = 1, tada je n(a, b) = a + b − 1. To znači da je rešenje uopštem slučaju n(a, b) = a + b − (a, b).32


9. Predlog za MMO, 1988. (Južna Koreja)Izračunati (f 1960 , f 1988 ), gde je f n niz Fibonačijevih brojeva (definisan sa f 1 =f 2 = 1 i f n+2 = f n+1 + f n za n 1).Rešenje. Sama rekurentna relacija Fibonačijevog niza izražava zavisnost članaf n+2 od f n i f n+1 . Ako pokušamo da izrazimo f n+i za i > 2 preko f n i f n+1 ,dobijamo redom:f n+3 = f n+2 + f n+1 = (f n+1 + f n ) + f n+1 = 2f n+1 + f n ,f n+4 = f n+3 + f n+2 = (2f n+1 + f n ) + (f n+1 + f n ) = 3f n+1 + 2f n ,f n+5 = f n+4 + f n+3 = (3f n+1 + 2f n ) + (2f n+1 + 1f n ) = 5f n+1 + 3f n ,. . .Sada se na rutinski način induktivno pokazuje da važif n+i = f i f n+1 + f i−1 f n (7)za sve i 2. Uvrštavajući i = (k − 1)n za k 2, sledif kn = f (k−1)n f n+1 + f (k−1)n−1 f n ,odakle se lako (ponovo indukcijom) dokazuje da važiza sve n, k ∈ N. Najzad, neposredno se dobija da jeza sve n ∈ N.Iz (7) dobijamo:f n | f kn (8)(f n+1 , f n ) = (f n , f n−1 ) = . . . = (f 2 , f 1 ) = 1 (9)f 1988 = f 1961 f 28 + f 1960 f 27 .Znači, ako je r = (f 1960 , f 1988 ), tada r | f 28 , pošto je na osnovu (9)(f 1961 , f 1960 ) = 1.Ali, kako je 1988 = 71 · 28 i 1960 = 70 · 28, to f 28 | r (zbog (8)), pa jer = f 28 = 317811.Komentar. Uopšte, može se pokazati da važi: (f s , f t ) = f (s,t) .33


10. Rumunija, 1991.Za dato c ∈ N, neka je n 0 najmanji prirodan broj takav da je 2 n 0da tada za sve n n 0 važia n | cb n ⇒ a | b,gde su a, b ∈ N.> c. DokazatiRešenje. Neka je p prost broj i neka su α, β, γ redom najviši stepeni kojim p delibrojeve a, b, c, za koje smo pretpostavili da a n | cb n (n n 0 ). Dakle, imamonα nβ + γ.S druge strane, kako je 2 n 0> c, to je γ < n 0 n. Otuda jenα < nβ + n = n(β + 1),pa dobijamo α < β + 1, tj. α β. Pošto je prost broj p u ovom razmatranju bioproizvoljan, sledi a | b.34


11. Predlog za MMO, 1984. (Velika Britanija)Dokazati da proizvod pet uzastopnih prirodnih brojeva ne može biti potpun kvadrat.Rešenje. Pretpostavimo suprotno: postoji pet uzastopnih prirodnih brojeva x, x+1,x+2, x+3, x+4 čiji je proizvod potpun kvadrat. Neka je p 5 prost broj. Najpre,primetimo da najviše jedan od navedenih brojeva može biti deljiv sa p. Ukoliko pakp | (x+i), 0 i 4, tada zbog naše pretpostavke najviši stepen kojim p deli x+imora biti paran.Ovo poslednje tvr - denje povlači da svaki od pet posmatranih brojeva mora bitioblika2 α 3 β t 2 0za neko t 0 ∈ N. Razmatrajući parnost brojeva α i β, zaključujemo da svaki oduočenih brojeva može biti prikazan u jednom od oblika t 2 , 2t 2 , 3t 2 , 6t 2 za odgovarajućet ∈ N. Po Dirihleovom principu, od tih pet brojeva postoje dva kojase redom mogu prikazati kao au 2 i av 2 za neke različite u, v ∈ N i neko fiksnoa ∈ {1, 2, 3, 6}. Ako je npr. u > v, imamoau 2 − av 2 = a(u 2 − v 2 ) (x + 4) − x = 4,što znači da je u 2 − v 2 4 a. To je moguće samo ako je a = 1, u = 2 i v = 1,pa su 1 i 4 medu - posmatranim brojevima, tj. u pitanju su baš 1, 2, 3, 4, 5. Medutim,-1 · 2 · 3 · 4 · 5 = 120, što nije potpun kvadrat. Dobijena kontradikcija pokazuje daje tvrdenje - zadatka tačno.35


12. Čehoslovačka, 1988.Postoji li 23-cifren prirodan broj takav da zamenom proizvoljne cifre nikada nedobijamo broj deljiv sa 11?Rešenje. Pretpostavimo da takav broj postoji; neka je tox =22∑k=0a k 10 k .Budući da je 10 2k ≡ 1(mod 11) i 10 2k−1 ≡ −1(mod 11), ako je r ostatak prideljenju x sa 11, imamo:r ≡11∑k=0∑11a 2k − a 2k−1 (mod 11).k=1Ako je a 2k r za neko k, tada a 2k možemo zameniti sa a ′ 2k = a 2k − r (što jecifra, jer je a ′ 2k a 2k), a u slučaju a 2k r − 2 sa a ′ 2k = 11 + a 2k − r (pri čemua ′ 2k 9 sledi iz a 2k r − 2), pa će tako dobijeni broj biti deljiv sa 11. Dakle, svecifre na parnim mestima broja x su jednake r − 1 (r ≠ 0).Analogno, ako je a 2k−1 9 − r, tada zamenom cifre a 2k−1 cifrom a ′ 2k−1 =a 2k−1 + r ( (9 − r) + r = 9), odnosno za a 2k−1 11 − r cifrom a ′ 2k−1 =a 2k−1 + r − 11 ( a 2k−1 ) dobijamo broj deljiv sa 11. Stoga su sve cifre broja x naneparnim mestima jednake 10 − r.Tada je, me - dutim,r ≡ 12(r − 1) − 11(10 − r) ≡ r − 1(mod 11),što je nemoguće. Dakle, takav broj ne postoji.36


13. Predlog za MMO, 1990. (S.S.S.R.)Naći sve prirodne brojeve n za koje su svi brojevi koji imaju u dekadnom prikazun − 1 cifru 1 i jednu cifru 7 prosti.Rešenje. Broj N koji se sastoji od n − 1 cifara 1 i jedne cifre 7 može se zapisati uoblikuN = A n + 6 · 10 k ,gde je A n broj koji je napisan sa n jedinica, a 0 k < n.Ako 3 | n, tada je zbir cifara broja N deljiv sa 3, pa stoga 3 | N, zbog čega Nnije prost.Sada posmatrajmo slučaj n 6. Kako brojevi A s redom za s = 1, 2, 3, 4, 5, 6daju ostatke 1, 4, 6, 5, 2, 0 pri deljenju sa 7, i kako jeA m+6 = A m 10 6 + A 6 ≡ A m + A 6 ≡ A m (mod 7),sledi da 7 | A l ako i samo ako 6 | l. Dalje, za k = 0, 1, 2, 3, 4, 5 broj 10 kdaje redom ostatke 1, 3, 2, 6, 4, 5, pa 6 · 10 k za iste vrednosti k redom daje ostatke6, 4, 5, 1, 3, 2. Odavde sledi zaključak: ako za neko n 6 važi A n ≡ r(mod 7) i3 ∤ n, tada možemo naći takvo k 5 da je 6 · 10 k ≡ −r(mod 7), pa onda7 | (A n + 6 · 10 k ),} {{ }Ntj. N nije prost.Znači, dovoljno je da proverimo vrednosti n = 2, 4, 5. Za n = 5 imamo11711 = 7 · 1673, a za n = 4 je 1711 = 29 · 59. S druge strane, za n = 2, brojevi17 i 71 su prosti. Prema tome, rešenje zadatka je n ∈ {1, 2}.37


14. Predlog za MMO, 1988. (Velika Britanija)Prirodan broj je dupli ako se njegov dekadni zapis sastoji od dva identična blokacifara. Dokazati da me - du duplim brojevima postoji beskonačno mnogo potpunihkvadrata.Rešenje. Neka je 2k broj cifara duplog broja koji se sastoji od dva bloka a. Tvr - denjezadatka je sada ekvivalentno tome da jednačinaa(10 k + 1) = b 2 (10)ima beskonačno mnogo rešenja (a, b, k) takvih da a ima tačno k cifara, tj.10 k−1 a < 10 k .Plan rešenja je sledeći: najpre ćemo za pogodne vrednosti k (kojih će bitibeskonačno mnogo) naći po jedno rešenje (a 1 , b 1 , k) jednačine (10) tako da a 1ima ne više od k cifara, a 1 < 10 k . Nakon toga, na osnovu primedbe da je tadarešenje i (n 2 a 1 , nb 1 , k) za sve n ∈ N, odabraćemo n 0 ∈ N tako da za a = n 2 0 a 1važe tražena ograničenja 10 k−1 a < 10 k .Naravno, ako posmatramo gornju jednačinu (10) bez ikakvih ograničenja, a =10 k + 1 jeste jedno očigledno rešenje. Kako bismo postigli prvi od postavljenihciljeva (rešenje u kome je a < 10 k ), dovoljno je da pronademo - beskonačno mnogovrednosti k za koje je broj 10 k + 1 deljiv potpunim kvadratom d 2 > 1. Tada će,naime,a 1 = 10k + 1d 2 , b 1 = 10k + 1,dpredstavljati rešenje razmatrane jednačine (10).Najpre, uočavamo da je10 3 + 1 = (10 + 1)(10 2 − 10 + 1) = 11 · 91,tj. važi 10 3 = −1 + 11c (za c = 91). Sada je ideja da se na osnovu ove relacijedobije stepen od 10 koji daje ostatak −1 pri deljenju sa 11 2 . To ćemo postići akostepenujemo poslednju jednakost sa 11, jer tada iz binomnog obrasca imamo10 33 = (−1 + 11c) 11 = −1 +za neko c ′ ∈ N. Otuda je( 111)11c −10 33(2m−1) ≡ −1 (mod 11 2 )38( 112)11 2 c 2 + . . . = −1 + 11 2 c ′


za sve m ∈ N, tj. 11 2 | (10 66m−33 + 1). To znači da je k = 66m − 33 i d =11 pogodan izbor, pa za svaku vrednost m imamo po jedno rešenje (10) kao uprethodnom pasusu.Najzad, preostaje da se prona - de n 0 ∈ N tako da važi10 k−1 n 2 10 k + 1011 2 < 10 k ,gde je k oblika 66m − 33. Očigledno, gornje nejednakosti su ekvivalentne sa10 k−1 (10 k + 1 n0) 2 10 k


15. Predlog za MMO, 1983. (Belgija)Neka je σ(n) zbir svih pozitivnih delitelja prirodnog broja n. Prirodan broj mzovemo jakim ako za sve 1 k < m važiσ(k)k< σ(m)m .Dokazati da postoji beskonačno mnogo jakih brojeva.Rešenje. Obeležimo a m = σ(m)m. Jasno, broj m je jak ako i samo ako je a k < a mza sve k < m. Sada je dovoljno dokazati da niz a n , n ∈ N, nema najveći element,pošto je tada lako izvesti da postoji beskonačno mnogo jakih brojeva. Naime, akoje broj m jak, nadimo - najmanje m ′ > m tako da je a m < a m ′. Tada je i broj m ′očigledno jak.Dakle, neka je n proizvoljan prirodan broj. Za svaki njegov delilac d važi da2d | 2n. Kako, osim toga, trivijalno važi 1 | 2n, dobijamo nejednakostσ(2n) 2σ(n) + 1.Odavde deljenjem sa 2n odmah sledi a 2n > a n , pa dobijamo traženi rezultat.40


16. S.A.D., 1978.Broj n je dobar ako se može predstaviti kao zbir (ne obavezno različitih) prirodnihbrojeva čiji je zbir recipročnih vrednosti jednak 1. Ako je poznato da su brojevi33, . . . , 73 dobri, dokazati da su svi brojevi 33 dobri.Rešenje. Neka je n dobar broj, pri čemu je za neke prirodne brojeve a i ispunjeno:a 1 + . . . + a k = n,1a 1+ . . . + 1 a k= 1.Tada jepa zbog 1 2 = 1 4 + 1 4 = 1 3 + 1 6 nizovi1+ . . . + 1 = 1 2a 1 2a k 2 ,(4, 4, 2a 1 , . . . , 2a k ) i (3, 6, 2a 1 , . . . , 2a k )imaju zbir recipročnih vrednosti jednak 1. Otuda dobijamo da su brojevi 2n + 8 i2n + 9 tako - de dobri. Me - dutim, 2 · 33 + 8 = 74 i 2 · 33 + 9 = 75, pa jednostavnomprimenom indukcije dobijamo, na osnovu date hipoteze, da su svi brojevi 33dobri.41


17. Čehoslovačka, 1988.Označimo sa S(n) zbir cifara prirodnog broja n u dekadnom zapisu.zovemo lošim ako se ne može predstaviti u oblikuBroj mm = n + S(n).Koliko ima loših brojeva — konačno ili beskonačno mnogo?Rešenje. Dokazaćemo da postoji beskonačno mnogo loših brojeva. Ideja rešenjase sastoji u tome da se konstruiše niz prirodnih brojeva a n , n 1, povezanihpogodnom rekurentnom relacijom koja će obezbediti da svi članovi niza budu loši.S obzirom na formulaciju zadatka, biće veoma pogodno da svi članovi niza imajurazličit broj cifara u dekadnom zapisu: tačnije, a n će imati n + 1 cifru.Najpre, neposredno se proverava da je broj 20 loš, pa definišemo a 1 = 20.Po - dimo od pretpostavke da smo uspeli da na - demo loše brojeve a 1 , . . . , a n−1 uskladu sa gornjim uslovima. Naš cilj je da konstruišemo (n + 1)-cifreni loš broja n . Da bismo to postigli, moramo da sprečimo da jednakost a n = k + S(k) budeispunjena za bilo koje k ∈ N. Razmotrićemo posebno slučajeve kada k ima ne višeod n cifara, odnosno kada k ima tačno n + 1 cifru.U prvom slučaju je k 10 n −1. Tada je S(k) 9n, pa sledi k+S(k) 10 n +9n − 1. Prema tome, ukoliko (odgovarajućom rekurentnom relacijom) obezbedimoda važia n > 10 n + 9n − 1, (11)tada će razmatrani slučaj biti nemoguć.Drugi slučaj nastupa kada je k 10 n . Tada možemo pisatik = c · 10 n + α,gde je c prva cifra od k i α 10 n − 1. Pri tome je S(k) = S(α) + c, tj.k + S(k) = c · 10 n + α + S(α) + c.Gornja jednakost sugeriše da se (za neko fiksno c ∈ {1, 2, . . . , 9}) za traženu rekurentnuvezu uzmea n = a n−1 + c(10 n + 1),jer tada iz pretpostavke da je a n = k + S(k) sledi da je a n−1 = α + S(α), štoomogućava da se zaokruži induktivni dokaz.Pokazaćemo da su za c = 1 ispunjeni svi traženi uslovi. Dakle, razmatramorekurentnu relacijua n = a n−1 + 10 n + 1, (12)42


pri čemu je a 1 = 20. Indukcijom dokazujemo da su svi članovi ovako definisanogniza loši.Prvo uočavamo da važi nejednakost (11). Naime, očigledno imamozbog čega je (za n 3):a r = a r−1 + 10 r + 1 > 10 r ,a n = a n−1 + 10 n + 1 > 10 n + 10 n−1 > 10 n + 9n − 1.Za n = 1, 2 se (11) proverava direktno. Dakle, ako bi bilo a n = k + S(k), tadamora biti k 10 n (štaviše, za slučaj k = 10 n se lako vidi da je nemoguć).S druge strane, važi a n < 2 · 10 n , što dobijamo neposrednom primenom indukcijena (12). Kako iz jednakosti a n = k + S(k) sledi k < a n , dobijamo da je prvacifra broja k jednaka 1. Sada iz gornjih razmatranja imamo da jek + S(k) = α + S(α) + 10 n + 1,gde je k = 10 n +α, pa pretpostavka da je a n = k+S(k) povlači a n−1 = α+S(α).Pošto je poslednja jednakost po induktivnoj pretpostavci nemoguća, sledi da se nia n ne može predstaviti u obliku k + S(k), tj. a n je loš broj.43


18. Ma - darska, 1989. (konkurs ”Kürschák József”)Neka je S(n) zbir cifara u dekadnom zapisu prirodnog broja n. Naći sve prirodnebrojeve M takve da važiS(kM) = S(M)za svako prirodno k M.Rešenje. Najpre, primetimo da je M = 1 trivijalno rešenje zadatka.Po - dimo najpre od pretpostavke da je M broj sa n cifara u dekadnom zapisu kojizadovoljava uslov zadatka. Neka je d njegova prva cifra; u tom slučaju, možemopisatiM = 10 n−1 d + m,gde je m < 10 n−1 . Tada je S(M) = d + S(m). Jasno je da (osim u slučaju n = 1)ne može biti M = 10 n−1 , pa možemo uzeti k = 10 n−1 + 1, čime dobijamo:odakle jekM = 10 2n−2 d + 10 n−1 (m + d) + m,S(kM) = S(m) + S((10 n−1 d + m + d)10 n−1 ) = S(m) + S(M + d).Znači, mora bitid = S(M + d).Podsetimo se da je d prva cifra broja M, pa je zato M + d > 10 n−1 d. Prema tome,ako bi bilo d < 9, tada bi prva cifra broja M + d bila ili d, ili d + 1. U oba slučajabismo imali S(M + d) > d. Zbog toga, d = 9. Štaviše, zbir cifara broja M + 9treba da je 9, što je moguće samo ako je M + 9 10 n . Lako se uočava da zapravosve cifre broja M moraju biti jednake 9, tj. M = 10 n − 1.S druge strane, uverićemo se da svi brojevi oblika 10 n −1 zadovoljavaju traženiuslov. Zaista, ako je M = 10 n − 1, tada je S(M) = 9n i za k M važi:S(kM) = S(k(10 n − 1)) = S((k − 1)10 n + (10 n − k)).Sada je (k − 1)10 n 10 n > 10 n − k 1 (ukoliko je k > 1), pa je zatoS(kM) = S(k − 1) + S(10 n − k).Možemo pisati 10 n − k kao (10 n − 1) − (k − 1), pa dobijamo:Tako, kao konačan rezultat imamo:S(10 n − k) = 9n − S(k − 1).S(kM) = 9n,što je i trebalo pokazati.Dakle, rešenja zadatka su M = 1 i M = 10 n − 1, n 1.44


19. Predlog za MMO, 1990. (Ma - darska)Neka je f 1 (x) kvadrat zbira cifara prirodnog broja x, aIzračunati f 1991 (2 1990 ).f n (x) = f 1 (f n−1 (x)).Rešenje. Najpre imamo sledeće grube ocene:pa je zatoOtuda je, dalje,i2 1990 < 8 700 < 10 700 ,f 1 (2 1990 ) < (9 · 700) 2 < 5 · 10 7 .f 2 (2 1990 ) (4 + 7 · 9) 2 < 5000f 3 (2 1990 ) < (4 + 3 · 9) 2 = 31 2 = 961.S druge strane, zbog 2 6 ≡ 1(mod 9) sledi2 1990 ≡ 2 4 ≡ 7(mod 9).Pošto svaki broj daje isti ostatak po modulu 9 kao i njegov zbir cifara, imamo da jef 1 (x) ≡ x 2 (mod 9) za sve x ∈ N, pa je zatoif 1 (2 1990 ) ≡ 7 2 ≡ 4(mod 9)f 2 (2 1990 ) ≡ 4 2 ≡ 7(mod 9).Najzad, imamo f 3 (2 1990 ) = n 2 , pri čemu je n < 31 iPrema tome,n ≡ f 2 (2 1990 ) ≡ 7(mod 9).f 3 (2 1990 ) ∈ {49, 256, 625}.U sva tri slučaja je, me - dutim, f 4 (2 1990 ) = 169, pa imamo za n 2:f 2n (2 1990 ) = 169, f 2n+1 (2 1990 ) = 256.Tako je konačan rezultat f 1991 (2 1990 ) = 256.45


20. MMO, 1984. (Poljska)Neka su a, b, c, d, a < b < c < d, neparni prirodni brojevi za koje važi:(1) ad = bc,(2) a + d = 2 k , b + c = 2 m , za neke prirodne brojeve k, m.Dokazati da je a = 1.Rešenje. Neposredno iz uslova zadatka sledi:0 < (d − b)(d − c) = d 2 − bd + bc − cd = d 2 − bd + ad − cd = d(a + d − b − c),zbog čega je a + d > b + c, odnosno k > m.Dalje, iz d = 2 k − a i c = 2 m − b imamo a(2 k − a) = b(2 m − b), tj.(b − a)(b + a) = 2 m (b − 2 k−m a). (13)Zbog k > m, b − 2 k−m a je neparan broj, pa je (b − a)(b + a) deljivo sa 2 m , ali nei sa 2 m+1 . Otuda je:b − a = 2 m 1p,b + a = 2 m 2q,pri čemu je m 1 + m 2 = m, m 1 , m 2 1 i p, q su neparni brojevi. Sabiranjem ovedve jednačine sledi2b = 2 m 1p + 2 m 2q.Pošto je b neparan broj, mora biti m 1 = 1 ili m 2 = 1. S druge strane, zbogb + c = 2 m i b < c važi2 m−1 > b > b − a = 2 m 1p 2 m 1,tj. m 1 < m − 1 i m 2 = m − m 1 > 1. Tako je m 1 = 1 i m 2 = m − 1, pa prethodnisistem postajeb − a = 2p,b + a = 2 m−1 q.Na osnovu toga je 2 m = b + c > b + a = 2 m−1 q, pa je q < 2, tj. q = 1.Uvrštavajući dobijene rezultate u (13), dobijamop = b − 2 k−m a.46


Iz prethodnog sistema je b = p+2 m−2 , odakle je p = p+2 m−2 −2 k−m a, odnosno2 k−m a = 2 m−2 .Kako je a neparan, ovo je moguće samo ukoliko je k = 2m − 2 i a = 1, što je itrebalo pokazati.Komentar. Iz gornjeg rešenja se lako vidi da su sve četvorke brojeva (a, b, c, d) kojezadovoljavaju uslove zadatka date sagde je m ∈ N.(1, 2 m−1 − 1, 2 m−1 + 1, 2 2m−2 − 1),47


21. Predlog za MMO, 1990. (Australija)Broj 9 se može predstaviti kao zbir dva uzastopna prirodna broja 9 = 4 + 5;štaviše, on se može zapisati kao suma bar dva uzastopna prirodna broja na tačnodva načina:9 = 4 + 5 = 2 + 3 + 4.Da li postoji prirodan broj koji se može predstaviti kao zbir 1990 uzastopnih prirodnihbrojeva i koji se može zapisati kao zbir bar dva uzastopna prirodna broja natačno 1990 načina?Rešenje. Pretpostavimo da broj N ima tražena svojstva. Prvi od dva uslova zadatkase može zapisati kaoN = m + (m + 1) + . . . + (m + 1989) = 995(2m + 1989),za neko m, stoga je N neparan broj, deljiv sa 5 i 199. Drugi uslov je da postojitačno 1990 parova prirodnih brojeva (n, k) za koje jeN = n + (n + 1) + . . . + (n + k) =(k + 1)(2n + k)2Prema tome, postoji tačno 1990 načina da prikažemo 2N u obliku:2N = (k + 1)(2n + k),pri čemu je k 1. Pošto je N neparan broj, to je jedan od ova dva faktora neparan,dok je drugi deljiv sa 2, ali ne i sa 4. Kako je očito k + 1 < 2n + k, sledi da svakaopisana faktorizacija broja 2N jednoznačno daje traženi par (n, k). Ako zapišemo:2N = 2 · 5 s 1199 s 2p s 33 . . . ps rr ,gde su p i prosti brojevi različiti od 2, 5 i 199, dobijamo da je broj delilaca broja 2Njednak(1 + 1)(s 1 + 1)(s 2 + 1) . . . (s r + 1).Tako, faktorizacija 2N = uv, u < v, ima (s 1 + 1)(s 2 + 1) . . . (s r + 1), a poštotrivijalna faktorizacija 2N = 1 · 2N daje k = 0, opisanih parova (n, k) ima(s 1 + 1)(s 2 + 1) . . . (s r + 1) − 1.Sada je (s 1 +1)(s 2 +1) . . . (s r +1) = 1991 = 11·181. Budući da zbog 5·199 | Nimamo s 1 , s 2 > 0, zaključujemo da je s 1 = 10, s 2 = 180 ili s 1 = 180, s 2 = 10,kao i s 3 = . . . = s r = 0. To znači da je N = 5 10 · 199 180 ili N = 5 180 · 199 10 , pasmo tako dobili jedina dva broja sa traženim osobinama.48.


22. MMO, 1981. (Belgija)(a) Za koje prirodne brojeve n 3 postoji skup od n uzastopnih prirodnih brojevatakav da je najveći element tog skupa delitelj najmanjeg zajedničkog sadržaocapreostalih brojeva?(b) Za koje prirodne brojeve n 3 postoji jedinstven skup sa gornjim svojstvom?Rešenje. (a) Označimo najveći od traženih brojeva sa m; tada tražimo prirodan brojn tako da važim | [m − n + 1, . . . , m − 2, m − 1].Očito, mora biti m n. Neka je k najveći ceo broj sa osobinom p k | m, pri čemuje p proizvoljan prost broj. Me - dutim, sada p k deli najmanji zajednički sadržalacbrojeva m − 1, m − 2, . . . , m − n + 1 ako i samo ako deli bar jedan od njih. Toznači da p k mora deliti bar jedan od brojeva 1, 2, . . . , n − 1, odnosno njihov najmanjizajednički sadržalac, koji ćemo označiti sa a(n). Zaključak je da je uslov,postavljen u zadatku, ekvivalentan sa postojanjem prirodnog broja m n takvogda m | a(n).Egzistencija bar jednog takvog broja m je ekvivalentna nejednakosti a(n) n,tj. a(n) > n − 1. Po definiciji broja a(n), (n − 1) | a(n). Pošto je n 3, imamoda (n − 2) | a(n). Zbog (n − 2, n − 1) = 1 sledi (n − 2)(n − 1) | a(n). Dakle,a(n) (n−2)(n−1), što je > (n−1) kad god je n 4. S druge strane, a(3) = 2,što znači da odgovor na pitanje (a) glasi: n 4.(b) Uslov ovog dela zadatka ekvivalentan je postojanju jedinstvenog prirodnogbroja m n tavkog da m | a(n). Pri tome, zbog prethodnog dela zadatka možemopretpostaviti da je n 4. Pošto tada m = a(n) zadovoljava tražene uslove, naš ciljse svodi na odre - divanje svih brojeva n 4 za koje su svi pravi delitelji od a(n)manji od n.Kao što smo to malopre videli, važi a(n) = (n − 1)(n − 2)b za neko b 1.Kako za n 4 imamo (n − 1)(n − 2) n, sledi da mora biti b = 1, tj. a(n) =(n − 1)(n − 2). Me - dutim, pošto (n − 3) | a(n) i (n − 3, n − 2) = 1, zaključujemoda (n − 3) | (n − 1). Drugim rečima, (n − 3) | 2, pa je n ∈ {4, 5}. Pošto je6 | 12 = a(5) i a(4) = 6, dobijamo da je odgovor na pitanje (b): n = 4.49


23. Predlog za MMO, 1974. (Poljska)Neka su a 1 , . . . , a k , b 1 , . . . , b k prirodni brojevi takvi da je (a i , b i ) = 1 za sve i, 1 mi k. Neka je m najmanji zajednički sadržalac brojeva b 1 , . . . , b k , a c i = a i b i,1 i k. Dokazati:(a 1 , . . . , a k ) = (c 1 , . . . , c k ).Rešenje. Neka je p prost broj koji deli bar jedan od datih brojeva a 1 , . . . , a k ,b 1 , . . . , b k . Neka su α i , β i redom najviši stepeni kojima p deli a i , odnosno b i . Tadaje najviši stepen kojim p deli m jednak µ = max j β j , dok je najviši stepen kojimp deli c i jednak γ i = α i + µ − β i . Imajući u vidu proizvoljnost prostog faktora p,tvr - denje zadatka je ekvivalentno jednakostiminiα i = min(α i + µ − β i ) = µ + min(α i − β i ). (14)iiPri tome zbog (a i , b i ) = 1 važi α i ≠ 0 ⇒ β i = 0.Radi kraćeg zapisa, definišimo δ i = α i − β i . S obzirom na uočeni odnoseksponenata α i i β i , važi{−βi ako α i = 0,δ i =α i ako α i ≠ 0.Razlikujemo dva slučaja. Ako postoji indeks i 0 tako da je α i0 = 0, tada jemin i α i = 0. S druge strane, min i δ i = − max i β i = −µ, pa tada i desna strana u(14) ima vrednost 0. U suprotnom slučaju, α i > 0 za sve 1 i k. Ali, tada jeβ i = 0 za sve 1 i k, pa je µ = 0 i δ i = α i za sve 1 i k. To znači da jejednakost (14) tačna i u ovom slučaju, pa je njen dokaz kompletan.50


24. Ma - darska, 1990. (konkurs ”Kürschák József”)Neka je p prost broj, a n prirodan broj. Dokazati da postoji najviše jedan pozitivanbroj d takav da važi d | pn 2 i da je n 2 + d potpun kvadrat.Rešenje. Neka jen 2 + d = m 2 , (15)i neka je z = (n, m). Tada imamo n = xz i m = yz, gde je (x, y) = 1, pa iz(15) sledi da je d = az 2 za neko a. Pri tome, az 2 = d | pn 2 = px 2 z 2 , pa a | px 2 .Nakon uvrštavanja u (15) i skraćivanja sa z 2 , dobijamoMnoženjem ove jednakosti sa p, sledix 2 + a = y 2 .px 2 + pa = py 2 .Kako a | px 2 , leva strana ove jednakosti je deljiva sa a, pa a | py 2 . Pošto je(x, y) = 1 (⇒ (x 2 , y 2 ) = 1), postoje celi brojevi α, β tako da je αx 2 + βy 2 = 1.Ali, tadaa | (αpx 2 + βpy 2 ) = p(αx 2 + βy 2 ) = p.Prema tome, a = 1 ili a = p.Prvi od dva navedena slučaja, a = 1, rezultuje jednačinom x 2 + 1 = y 2 , odakleje x = 0 i n = xz = 0, što je nemoguće. Dakle, moguć je samo drugi slučaj,a = p. Tada važi x 2 + p = y 2 , odnosno p = y 2 − x 2 = (y − x)(y + x), što povlačiy − x = 1 i y + x = p, tj.y = p + 12, x = p − 12Otuda je n = xz = 1 2(p − 1)z.Znači, egzistencija broja d sa traženim osobinama povlači da 1 2(p − 1) | n.S druge strane, ukoliko ova deljivost važi, tada imamo jedinstveno d za koje suispunjeni traženi uslovi, naimeTime je traženo tvr - denje dokazano.( n) ( ) 2 2n 2d = pz 2 = p = p .x p − 1.51


25. Predlog za MMO, 1995. (Rumunija)Neka je k dati prirodan broj. Dokazati da postoji beskonačno mnogo potpunihkvadrata oblika 2 k n − 7.Rešenje. Dokažimo najpre da za svaki prirodan broj k postoji prirodan broj a k saosobinoma 2 k ≡ −7(mod 2k ).Primetimo da izbor a k = 1 zadovoljava traženi uslov za k 3. Za k 4,po - dimo od pretpostavke a 2 k ≡ −7(mod 2k ). Sada očito imamo dve mogućnosti:ilia 2 k ≡ −7(mod 2k+1 ),a 2 k ≡ 2k − 7(mod 2 k+1 ).U prvom slučaju definišimo a k+1 = a k , a u drugom a k+1 = a k + 2 k−1 . Pošto jea k neparno, u potonjem slučaju sledi:a 2 k+1 = a2 k + 2k a k + 2 2k−2 ≡ a 2 k + 2k a k ≡ a 2 k + 2k ≡ −7(mod 2 k+1 ),koristeći induktivnu pretpostavku.Najzad, primetimo da niz a k nije ograničen, pošto mora biti a 2 k 2k − 7,što znači da posmatrani niz ima beskonačno mnogo različitih vrednosti. Otudadobijamo traženi rezultat, pošto za m k imamo a 2 m ≡ −7(mod 2 k ) i možemodefinisatin = a2 m + 72 k .52


26. S.A.D., 1982.Dokazati da postoji prirodan broj k takav da je broj 2 n k + 1 složen za sve n ∈ N.Rešenje. Neka je n = 2 r p, gde je p neparan broj. Ako je broj M takav da jek ≡ 1(mod M), tada se k može ”izbrisati” iz izraza 2 n k + 1 (mod M), tj. imamo2 n k + 1 ≡ 2 n + 1 = 2 2rp + 1 = (2 2r ) p + 1 = (2 2r + 1)A (mod M)za neki prirodan broj A. Odavde očigledno sledi da ako za M odaberemo baš2 2r + 1 (ili bilo koji njegov delitelj), tada iz datih pretpostavki sledi da je 2 n k + 1deljivo sa 2 2r + 1.Poznato je da su za 0 r 4, brojevi F r = 2 2r + 1 prosti. (Inače, brojevioblika F r su Fermaovi, a u slučaju da je takav broj prost, on se zove Fermaov prostbroj. Malopre navedeni brojevi su ujedno i jedini do sada poznati Fermaovi prostibrojevi.) Me - dutim, kao što je to još 1732. godine pokazao L.Ojler, F 5 je složen:najmanji prost faktor mu je 641.Ideja rešenja se sastoji u tome da se k odredi uz pomoć kineske teoreme, takoda zadovoljava pogodan sistem linearnih kongruencija. Taj sistem kongruencija ćebiti ”podešen” tako da je za sve n broj 2 n k + 1 deljiv ili nekim od F 0 , . . . , F 4 , ili sa641, ili sa G = F 5 /641. Razlikovanje slučajeva se vrši po najvišem stepenu dvojkekoji deli n. Naime, već smo videli da za 0 r 4 imamo da ako važi n = 2 r p zaneki neparan broj p i k ≡ 1(mod F r ), tada F r | (2 n k + 1). Tako - de, ako je n = 2 5 pza neki neparan broj p i k ≡ 1(mod 641), tada 641 | (2 n k + 1).Prema tome, preostaje da se razmotri slučaj kada je n = 2 6 q za neko q ∈ N.Tada imamo2 n k + 1 = k(2 n − 1) + (k + 1) = k((2 64 ) q − 1) + (k + 1) == k(2 64 − 1)B + (k + 1) = kF 5 (F 5 − 2)B + (k + 1)za neki prirodan broj B, pa ukoliko je k ≡ −1(mod G), dobijamo da važi G |(2 n k + 1) (pošto G | F 5 ).Rezimirajući prethodna razmatranja, tražimo prirodan broj k koji zadovoljavak ≡ 1(mod F r ), 0 r 4, k ≡ 1(mod 641), k ≡ −1(mod G).Pošto su brojevi F 0 , . . . , F 4 , 641 prosti, a F 5 nije deljivo sa 641 2 (što se neposrednoproverava), moguća je primena kineske teoreme o ostacima. Štaviše, gornjisistem ima proizvoljno veliko rešenje k, pa tako i rešenje koje zadovoljava k max(F 0 , F 1 , F 2 , F 3 , F 4 , 641, G). Zbog toga je2 n k + 1 > max(F 0 , F 1 , F 2 , F 3 , F 4 , 641, G)53


za sve n ∈ N. S druge strane, pokazali smo da je broj oblika 2 n k + 1 (za na opisaninačin odabrano k) uvek deljiv bar jednim od F 0 , F 1 , F 2 , F 3 , F 4 , 641, G. Zato je2 n k + 1 složen broj za sve n ∈ N, što je i trebalo dokazati.54


27. Rumunija, 1978.Neka su m i n prirodni brojevi sa osobinom da za sve prirodne brojeve k važi(11k − 1, m) = (11k − 1, n). Dokazati da tada za neki ceo broj s važi m n = 11s .Rešenje. Neka je m = 11 a p, n = 11 b q, pri čemu je a, b 0 i brojevi p, q nisudeljivi sa 11. Dokazaćemo da je p = q, odakle sledi tvr - denje zadatka.Kako je (p, 11) = 1, po kineskoj teoremi o ostacima postoji prirodan broj xkoji zadovoljava:x ≡ 0(mod p), x ≡ −1(mod 11).Ali, tada je x = 11k − 1 za neki prirodan broj k, pa je:p = (x, 11 a p) = (11k − 1, m) = (11k − 1, n) = (x, 11 b q) = (x, q) q.Potpuno analogno možemo pokazati da je q p, stoga sledi zaključak p = q.55


28. Rumunija, 1991.Naći sve aritmetičke progresije u kojima su za sve n ∈ N zbirovi prvih n članovapotpuni kvadrati.Rešenje. Ako je a prvi član aritmetičke progresije, a d njena razlika, tada imamoformulu za sumu prvih n članova progresije:S n = 1 n(2a + (n − 1)d).2Specijalno, imamo S 1 = a i S 4 = 2(2a + 3d), odakle je a = b 2 i d = 2d 1 za nekeprirodne brojeve b i d 1 , pa sledi:S n = n(b 2 + (n − 1)d 1 ) = n(nd 1 + (b 2 − d 1 )).Za proizvoljan prost broj p iz p | S p sledi p 2 | S p , što na osnovu gornje jednakostipovlači p | (b 2 − d 1 ). To je moguće ako i samo ako je b 2 − d 1 = 0, tj. d = 2b 2 .S druge strane, ako su članovi aritmetičke progresije oblika (2n−1)b 2 , gde je bproizvoljan prirodan broj, tada su odgovarajuće sume S n = n 2 b 2 , što znači da smonašli sve tražene progresije.56


29. MMO, 1991. (Rumunija)Neka je n > 6 i neka su a 1 < a 2 < . . . < a k svi prirodni brojevi manji od ni uzajamno prosti sa n. Dokazati: ako je niz a i aritmetička progresija, tada je nprost broj ili stepen dvojke.Rešenje. Na početku uočavamo da je a 1 = 1 i a 2 = p, gde je p najmanji prostbroj koji ne deli n. Zato je razlika posmatrane aritmetičke progresije jednaka p − 1.Tako - de, a k = n − 1.Ako je n neparan broj, tada je a 2 = 2, pa je uočena progresija zapravo 1, 2, . . . ,n − 1, odakle sledi da je broj n prost.Ukoliko je n paran, tada je p 3, pa razmatramo dva slučaja. Ako je p = 3,tada je reč o progresiji 1, 3, . . . , n − 1, što znači da je n uzajamno prost sa svakimneparnim brojem manjim od n, što je moguće samo ako je n = 2 m za neki prirodanbroj m. Ako je p > 3, sledi da 3 | n. Tada zbogn − 2 = (n − 1) − 1 = a k − a 1 = (p − 1)(k − 1)važi (p−1) | (n−2), pa ako je q prost broj i q | (p−1), tada q | (n−2). Me - dutim,q < p, pa q | n. Otuda je q | 2, tj. q = 2, kao i p − 1 = 2 s za s 2, odnosnop = 2 s + 1. Pošto je p prost broj, to je s = 2 t , p = 2 2t + 1 za t 1. Ali, sadaimamo:a 3 = 1 + 2(p − 1) = 2p − 1 = 2 2t +1 + 1,pa 3 | a 3 . Zato 3 | (a 3 , n) = 1. Kontradikcija.57


30. Ma - darska, 1991.Neka je n ∈ N. Dokazati da postoje različiti brojevi a, b, c ∈ N tako da jei a | bc.n 2 < a, b, c < n 2 + n + 3 √ nRešenje. Pošto za tražene brojeve mora da važi a | bc, postoje x, y tako da jea = xy, pri čemu x | b i y | c. Budući da nejednakosti date u zadatku sugerišu dabrojevi a, b, c treba da budu relativno ”bliski”, potražićemo ih u obliku:a = xy, b = x(y + 1), c = (x + 1)y,pri čemu je x < n < y (odakle je a < b < c). Zbog toga, pišimo x = n − k,y = n + l za neke prirodne brojeve k, l.Kako bismo pronašli brojeve x, y sa željenim osobinama, potrebno je i dovoljnoda bude xy = (n−k)(n+l) > n 2 i (x+1)y = (n−k+1)(n+l) < n 2 +n+3 √ n.Nakon kraćeg sre - divanja, ovi uslovi postaju(l − k)n − kl > 0, (l − k)n − kl + l < 3 √ n.Naravno, ovde će izbor k, l zavisiti od n. Iz gornjih nejednakosti vidimo da broj(l−k)n−kl treba da bude pozitivan, ali, s druge strane, ne previše velik. Zbog togaje naš cilj da razlika l − k bude što je manja moguća. Kako iz prve nejednakostiočigledno sledi da mora biti l − k > 0, intuitivno je opravdano da pokušamo dapostignemo da bude l − k = 1, tj. l = k + 1. Ako ovo uvrstimo u gornji sistemnejednakosti, dobijamo uslove k(k+1) < n i n−k 2 +1 < 3 √ n, što je ekvivalentnosa k 2 > n − 3 √ n + 1.Zbog ovih uslova, pogodno će biti da za k uzmemo najveću vrednost za koju jeispunjena nejednakost k(k + 1) < n. Drugim rečima, treba da budek(k + 1) < n (k + 1)(k + 2).Kao što smo već videli, nejednakost k(k + 1) < n je ekvivalentna sa a > n 2 , papreostajemo da pokažemo da za opisani izbor k i l imamo c < n 2 + n + 3 √ n, tj.k 2 > n − 3 √ n + 1. Zaista, ako pretpostavimo da ova nejednakost nije ispunjena,tada jek 2 n − 3 √ n + 1 < ( √ n − 1) 2 ,odakle je k < √ n − 1. Stoga, zaključujemo:(k + 1)(k + 2) = k 2 + 3k + 2 < (n − 3 √ n + 1) + 3( √ n − 1) + 2 = n,što je kontradikcija sa izborom k.58


31. Ma - darska, 1991.Za prirodne brojeve a, b, c, d važi ad = bc i a < b < c < d. Dokazati da postojiprirodan broj n tako da je a < n 2 < d.Rešenje. Pretpostavimo suprotno tvr - denju zadatka; tada postoji prirodan broj ntako da je n 2 a i d (n + 1) 2 . Otuda mora biti d − a 2n + 1. S druge strane,iz (c − b) 2 > 0 i uslova zadatka sledi(c + b) 2 − (c − b) 2 = 4bc = 4ad = (d + a) 2 − (d − a) 2 .Zbog d − a > c − b > 0 važi (d − a) 2 > (c − b) 2 , odakle iz gornjih veza imamo(d + a) 2 > (c + b) 2 , tj. d + a > c + b, kao i:(d − a) 2 = (d + a) 2 − (c + b) 2 + (c − b) 2 > (d + a) 2 − (c + b) 2 == (d − c − b + a)(d + c + b + a). (16)Po prethodnoj primedbi, d − c − b + a > 0, odnosno d − c − b + a 1. Tako - de,važi d + c + b + a > 4n 2 , pa jed − a > 2n.Zaključujemo da je jedina mogućnost d − a = 2n + 1, a = n 2 , d = (n + 1) 2 ,bc = n 2 (n + 1) 2 .Ako je n = 1, tada je a = 1, d = 4, odakle je b = 2, c = 3 i bc ≠ 4, pa je tajslučaj isključen. S druge strane, za n 2 važi nejednakost(2n + 1) 2 = 4n 2 + 4n + 1 < 8n 2 = 2 · 4n 2 ,pa uočavamo da (zbog (16)) mora biti d − c − b + a = 1, tj.c + b = (n + 1) 2 + n 2 − 1 = 2n(n + 1) = 2 √ ad = 2 √ bc.Transformacijom gornje jednakosti sledi ( √ c − √ b) 2 = 0, odnosno b = c. Kontradikcija.59


32. Predlog za MMO, 1978. (Čehoslovačka)Naći sve prirodne brojeve n < 1978 sa osobinom: ako je prirodan broj m, 1 8).Dakle, preostaje da se razmotre slučajevi k ∈ {0, 1, 2, 3, 4}.Za k = 0 imamo q = p 1 = 2, pa je n < 4 i 2 ∤ n, odnosno n ∈ {1, 3}. Jasno,odmah se vidi da su ovo rešenja zadatka.Za k = 1 je q = 3, pa n zadovoljava n < 9 i 2 | n, 3 ∤ n. Ponovo se lakoproverava da sva tri ”kandidata” 2, 4, 8 predstavljaju rešenja zadatka.Za k = 2 važi q = 5, odakle je n < 25, 6 | n i 5 ∤ n, tj. n ∈ {6, 12, 18, 24}.Direktnom proverom dobijamo da sva četiri navedena broja zadovoljavaju uslovezadatka.Za k = 3 sledi q = 7, pa mora biti n < 49, 30 | n i 7 ∤ n. Jedini brojsa navedenim osobinama je n = 30, za koji se direktno proverava da zadovoljavauslove zadatka.Najzad, za k = 4 je q = 11, što povlači da je n < 121 i 2 · 3 · 5 · 7 = 210 | n.Jasno, takav prirodan broj ne postoji.Prema tome, skup rešenja zadatka je{1, 2, 3, 4, 6, 8, 12, 18, 24, 30}.Komentar. Primetimo da je uslov n < 1978 suvišan u zadatku. Zvanično rešenjezadatka prezentirano žiriju MMO 1978. je nešto duže i koristi ovaj uslov.60


33. Predlog za MMO, 1985. (S.S.S.R.)Predstaviti broj 5 1985 − 1 kao proizvod tri prirodna broja od kojih je svaki veći od5 100 .Rešenje. Najpre, primetimo da je 1985 = 5 · 397. Zbog toga, označimo x = 5 397 .Sada je razmatrani broj x 5 − 1. Očigledno, imamo faktorizacijux 5 − 1 = (x − 1)(x 4 + x 3 + x 2 + x + 1).Naš cilj je da izraz u drugoj zagradi predstavimo kao razliku dva kvadrata, pričemu treba imati na umu da je x neparan stepen od 5, zbog čega je 5x potpunkvadrat (5x = 5 398 = (5 199 ) 2 ). Imajući to u vidu, tražićemo cele brojeve a, b, ctako da važix 4 + x 3 + x 2 + x + 1 = (x 2 + ax + 1) 2 − 5x(bx + c) 2 . (17)Razvijanjem izraza sa desne strane, dobijamox 4 + (2a − 5b 2 )x 3 + (a 2 + 2 − 10bc)x 2 + (2a − 5c 2 )x + 1,odakle nije teško videti da će (17) važiti za a = 3, b = c = 1. Prema tome, sledix 5 − 1 = (x − 1) [ (x 2 + 3x + 1) 2 − 5x(x + 1) 2] .Kao što je već rečeno, izraz u uglastoj zagradi je razlika kvadrata, pa se on možefaktorisati. Neposredno se proverava da su sva tri ovako dobijena faktora od x 5 − 1veća od 5 100 .61


34. Predlog za MMT, Mersch, Luksemburg, 1980. (Belgija)Neka je p(x) polinom sa celim koeficijentima takav da je p(0) = p(1) = 1. Nekaje a 1 proizvoljan ceo broj. Definišemo niz a 1 , a 2 , . . . , a n , . . . tako da za sve n 1važia n+1 = p(a n ).Dokazati da je svaki par različitih elemenata ovog niza uzajamno prost.Rešenje. Posmatrajmo elemente a m i a k datog niza, pri čemu je m > k. Ako jed = m − k, tada imamo da jea m = a k+d = p d (a k ) = p(p(. . . (p(a } {{ } k )) . . .)).dPri tome, p d (x) je tako - de polinom sa celim koeficijentima.Kako je p(0) = 1, sledi da je slobodni član polinoma p(x) jednak 1. Me - dutim,p 2 (0) = p(p(0)) = p(1) = 1, pa to isto važi i za polinom p 2 (x). Ali, sadase indukcijom lako dokazuje da je slobodni član polinoma p r (x) jednak 1 za sver 1, budući da iz pretpostavke da je p r (0) = 1 sledip r+1 (0) = p(p r (0)) = p(1) = 1.Za svaki polinom f(x) sa celim koeficijentima postoji polinom q(x) ∈ Z[x]tako da je f(x) = xq(x) + f(0). Specijalno, postoji q d (x) ∈ Z[x] tako da jeOtuda dobijamo da važip d (x) = xq d (x) + p d (0) = xq d (x) + 1.a m = p d (a k ) = a k q d (a k ) + 1 = a k Q + 1,gde je Q = q d (a k ) ceo broj. Iz gornje jednakosti sada neposredno sledi tvr - denjezadatka.62


35. Predlog za MMO, 1989. (Bugarska)Neka je n > 1. Dokazati da sledeći polinom nema racionalne nule:p n (x) =n∑k=0x kk! .Rešenje. Pretpostavimo da je α = q rracionalna nula datog polinoma, (q, r) = 1.Tada važin∑ n!( q) k= n!pn (α) = 0,k! rk=0odakle množenjem sa r n sledin−1∑q n n!+ rk! qk r n−1−k = 0.k=0Znači, r | q n , pa mora biti r = 1 (tj. α je ceo broj). Zbog toga, gornju jednakostmožemo pisati u oblikun−1∑q n n!+k! qk = 0. (18)k=0Kako je za 0 k n − 1 broj n!k!deljiv sa n, zaključujemo da n | q n . Toznači da za proizvoljan prost faktor p broja n imamo p | q n , odakle p | q. Ideja upreostalom delu rešenja je da dobijemo kontradikciju posmatrajući stepene kojimp deli pojedine sabirke na levoj strani jednakosti (18).Imajući to u vidu, označimo sa m najviši stepen kojim p deli n!. Pošto p k | q k ,dobijamo da p m+k | n!q k za sve 1 k n. S druge strane, stepen kojim p deli k!je po Ležandrovoj formuli jednak:⌊ ⌋ ⌊ ⌋ ⌊ ⌋k k k+p p 2 + . . . +p s s∑j=1kp j s∑j=1k2 j < k.Stoga p k ∤ k!, tj. stepen kojim p deli k! nije veći od k − 1. Zbog toga,∣( ) ∣∣∣ n!p m+1 k! qk ,gde je 1 k n. Posmatranjem člana u relaciji (18) koji odgovara k = 0,zaključujemo da p m+1 | n!. Me - dutim, to je kontradikcija sa izborom broja m.63


36. MMO, 1987. (S.S.S.R.)Neka je f(x) =⌊√x 2 +⌋x + p, gde je p ∈ N. Dokazati: ako je f(x) prost broj za svex ∈ {0, 1, . . . , p3}, tada je f(x) prost broj za sve x ∈ {0, 1, . . . , p − 2}.Rešenje. Zadatak će biti rešen ako dokažemo sledeće tvr - denje: ako postoji x ∈{0, . . . , p − 2} takav da je f(x) složen i ako je y najmanji broj iz skupa {0, . . . ,p − 2} sa tom osobinom, tada je y 4y 2 + y,√tj. p > 3y 2 , pa je y < p3, što je i trebalo dokazati.64


37. Predlog za MMO, 1992. (Irska)Dat je polinomf(x) = x 8 + 4x 6 + 2x 4 + 28x 2 + 1.Neka je p > 3 prost broj takav da postoji z ∈ N za koji p | f(z). Dokazati dapostoje celi brojevi z 1 , . . . , z 8 takvi da za g(x) = (x − z 1 ) . . . (x − z 8 ) važi da susvi koeficijenti polinoma f(x) − g(x) deljivi sa p.Rešenje. Za početak, nameće se očigledna smena X = x 2 . Na taj način, imamopolinomf 1 (X) = X 4 + 4X 3 + 2X 2 + 28X + 1.Naredni korak u ”pojednostavljivanju” ovog polinoma je da eliminišemo (baš kao uklasičnoj algebri, tj. teoriji algebarskih jednačina) kubni član. To postižemo putemsmene y = X + 1, pa dobijamoh(y) = y 4 − 4y 2 + 32y − 28.Ideja rešenja je da (po modulu p) faktorišemo ovaj polinom. Naš prvi cilj će biti dapokažemo da dati uslovi omogućavaju da se h(y) predstavi kao razlika kvadrata.Naime, primetimo da važih(y) = (y 2 + 2) 2 − 2(2y − 4) 2 .Dokazaćemo da pod datim pretpostavkama postoji kvadrat celog broja koji dajeostatak 2 pri deljenju sa p (tj. da je 2 kvadratni ostatak po modulu p).Kako p | f(z) = h(t) (gde smo označili t = z 2 + 1), sledi(t 2 + 2) 2 ≡ 2(2t − 4) 2 (mod p).Sada 2t − 4 nije deljivo sa p, jer bi u suprotnom bilo ili p = 2, ili t ≡ 2(mod p).U drugom slučaju bismo imali (iz gornje kongruencije) da p | 6 2 , što je suprotnopretpostavci p > 3. Dakle, postoji u tako da je u(2t − 4) ≡ 1(mod p), odakle jeOznačimo a = u(t 2 + 2). Sada imamo:[u(t 2 + 2)] 2 ≡ 2(mod p).h(y) ≡ (y 2 + 2) 2 − (2a(y − 2)) 2 (mod p).Nakon razvijanja gornje razlike kvadrata, jedan od faktora jey 2 + 2 − 2a(y − 2) = y 2 − 2ay + a 2 − a 2 + 4a + 2 = (y − a) 2 + (2 + 4a − a 2 ),65


što je po modulu p kongruentno sa (y − a) 2 + 4a. Na potpuno analogan način,drugi faktor je (mod p) jednak (y + a) 2 − 4a, pa je takoh(y) ≡ [(y − a) 2 + 4a][(y + a) 2 − 4a] (mod p).Sada za gornje faktore primenjujemo prethodnu ideju, pokazujući da su obabroja a, −a kvadratni ostaci po modulu p. Kako je h(t) deljivo sa p, jedan odbrojeva(t − a) 2 + 4a, (t + a) 2 − 4aje deljiv sa p. Pošto p ≠ 2, bar jedan od brojeva a, −a je kongruentan nekomkvadratu (mod p), tj. postoji b tako da jeb 2 ≡ ±a(mod p).U oba slučaja, zaključujemo da za neki ceo broj b važi:i p deli brojh(y) ≡ [(y − b 2 ) 2 + 4b 2 ][(y + b 2 ) 2 − 4b 2 ] (mod p), (19)[(t − b 2 ) 2 + 4b 2 ][(t + b 2 ) 2 − 4b 2 ] = [(t − b 2 ) 2 + 4b 2 ](t + b 2 − 2b)(t + b 2 + 2b) == [(t − b 2 ) 2 + (2b) 2 ][z 2 + (b − 1) 2 ][z 2 + (b + 1) 2 ].(Podsetimo se da je t = z 2 + 1.) Sledi da p deli zbir dva kvadrata; štaviše, u pitanjuje zbir dva kvadrata od kojih nijedan nije deljiv sa p (dovoljno je uočiti da p ∤ z pouslovima zadatka, kao i da p ∤ 2b jer je p > 3 i p | b bi povlačilo p | a, što nijemoguće po izboru a). Prema tome, za neke cele brojeve v, w takve da p ∤ v, p ∤ wimamo v 2 ≡ −w 2 (mod p). Ako je c ceo broj takav da je cw ≡ 1(mod p), tada je(cv) 2 ≡ −(cw) 2 ≡ −1(mod p), (20)tj. −1 kvadratni ostatak (mod p). Uvrštavajući ovo u (19), h(y) se (mod p) razlažena linearne faktore:h(y) ≡ (y − b 2 − 2bcv)(y − b 2 + 2bcv)(y + b 2 − 2b)(y + b 2 + 2b).Vraćajući smenu y = x 2 + 1 i vodeći računa o (20), sledi da je[x 2 − (b + cv) 2 ][x 2 − (b − cv) 2 ][x 2 − ((b − 1)cv) 2 ][x 2 − ((b + 1)cv) 2 ]razlaganje polinoma f(x) po modulu p, pa osam brojeva ±(b + cv), ±(b − cv),±(b − 1)cv, ±(b + 1)cv zadovoljava tražene uslove.66


38. Predlog za MMO, 1987. (Jugoslavija)Neka je k 2 proizvoljan prirodan broj. Dokazati da postoji iracionalan broj r ktakav da za svaki prirodan broj n važi:⌊rk n ⌋ ≡ −1(mod k).Rešenje. Budući da su najjednostavnije iracionalnosti kvadratne, osnovna ideja jeda se broj r k konstruiše kao rešenje kvadratne jednačine. Naime, posmatrajmojednačinut 2 − at + b = 0,gde je a, b ∈ N, pri čemu su njena rešenja r, s realna. Tada je r + s = a i rs = b,pa se indukcijom lako dokazuje da je za sve n ∈ N broj r n + s n ceo — primenjujese jednakost r n+1 + s n+1 = (r + s)(r n + s n ) − rs(r n−1 + s n−1 ), tj. ako označimoX n = r n + s n , tada je X n+1 = aX n − bX n−1 . Ukoliko su r, s pozitivni, i pri tomje još s < 1, tada je s n < 1 za sve n ∈ N, pa sledi da jer n + s n = ⌊r n ⌋ + 1.Drugim rečima, ⌊r n ⌋ = X n − 1, pa se naš cilj svodi na to da obezbedimo da svibrojevi X n budu deljivi sa k. Očigledno, to će biti tačno ako su oba koeficijentaa, b deljiva sa k.Prema tome, posmatraćemo kvadratnu jednačinu oblikax 2 − kpx + kq = 0, (21)a parametre p, q ∈ N ćemo odrediti tako da njena rešenja budu pozitivna, iracionalna,i da jedno njeno rešenje s bude < 1. Kao što smo iz prethodnog videli,tada se njeno drugo rešenje može uzeti za r k .Da bi rešenja od (21) uopšte bila realna, potrebno je da bude(kp) 2 − 4kq > 0,dok se pozitivnost rešenja i s < 1 obezbe - duje uslovom0 < kp − √ (kp) 2 − 4kq2Ove dve relacije zajedno su ekvivalentne sa uslovimap 2 > 4qk , p > q.67< 1.


Očito, ove uslove je moguće postići. Iracionalnost rešenja uočenog kvadratnogtrinoma dobijamo ako uzmemo npr. q = k. Naime, tada je diskriminanta jednakak 2 (p 2 − 4), što ne može biti potpun kvadrat, jer bi u suprotnom bilo4 = p 2 − t 2 = (p − t)(p + t)za neki prirodan broj t, što je nemoguće. Sada je dovoljno uzeti proizvoljno p k + 1.68


39. Predlog za MMO, 1979. (Belgija)Dokazati: ako prirodan broj A nije potpun kvadrat, tada postoji prirodan broj ntako da je A = ⌊ n + √ n + 1 2⌋.Rešenje. Pretpostavimo da se broj A 0 ne može prikazati u traženom obliku. Toznači da interval [A 0 , A 0 + 1) ne sadrži nijedan član niza n + √ n + 1 2, pa postojitakvo n 0 ∈ N da važin 0 + √ n 0 + 1 2 < A 0,kao iA 0 + 1 n 0 + 1 + √ n 0 + 1 + 1 2 .Objedinjujući ove dve nejednakosti, dobijamo:√n0 < A 0 − n 0 − 1 2 √ n 0 + 1,pa kvadriranjem imamo:n 0 < (A 0 − n 0 ) 2 − (A 0 − n 0 ) + 1 4 n 0 + 1,odnosno, dodavanjem (A 0 − n 0 ):A 0 < (A 0 − n 0 ) 2 + 1 4 A 0 + 1.Budući da su brojevi A 0 i n 0 prirodni, zaključak koji sledi iz ovih nejednakosti jeA 0 = (A 0 − n 0 ) 2 .Dakle, A 0 je kvadrat prirodnog broja, pa je time tvrdenje - zadatka dokazano.69


40. Predlog za MMO 1989. (Rumunija)Dokazati da niz brojeva ⌊n √ 2⌋, n ∈ N, sadrži beskonačno mnogo potpunih kvadrata.Rešenje. Uslov ⌊n √ 2⌋ = k 2 ekvivalentan je sa k 2 n √ 2 < k 2 + 1, odnosnok 4 2n 2 < (k 2 + 1) 2 . (22)Naš zadatak je da prona - demo beskonačno mnogo parova prirodnih brojeva (n, k)koji zadovoljavaju ovaj uslov. Zahtev da dva broja, od kojih je jedan kvadrat (k 4 ),a drugi dvostruki kvadrat (2n 2 ), budu ”bliski” asocira na Pelovu jednačinu2x 2 − y 2 = 1.Ako ovu jednačinu zapišemo u obliku 2x 2 = y 2 +1 i pomnožimo je sa y 2 dobićemo2(xy) 2 = y 4 + y 2 . Me - dutim, za svaki ceo broj y važiy 4 y 4 + y 2 < y 4 + 2y 2 + 1 = (y 2 + 1) 2 ,što znači da za svako rešenje (a, b) gornje jednačine, brojevi n = ab, k = b zadovoljavaju(22). Zato preostaje da pokažemo da posmatrana jednačina ima beskonačnomnogo rešenja.Uočimo da je 1 = 1 m = ( √ 2+1) m ( √ 2−1) m . Lako se pokazuje (indukcijom,ili primenom binomne formule) da tada postoje prirodni brojevi x m , y m tako da jeMe - dutim, ako je m neparno, tada je( √ 2 + 1) m = x m√2 + ym .( √ 2 − 1) m = x m√2 − ym ,Množenjem ove dve jednakosti dobijamo 2x 2 m = ym 2 + 1, pa sledi da je par(x m , y m ) za neparne vrednosti m jedno od željenih rešenja. Očigledno, svi oviparovi su različiti, pa je zadatak rešen, pošto je⌊x m y m√2⌋ = y2m .70


41. Predlog za MMO 1985. (Rumunija)Dokazati da niz brojeva ⌊n √ 2⌋, n ∈ N, sadrži beskonačno mnogo stepena dvojke.Rešenje. Traženo tvr - denje se može iskazati i na sledeći način: postoji beskonačnomnogo prirodnih brojeva n za koje postoji prirodan broj k tako da jeodnosno,Me - dutim, interval2 k n √ 2 < 2 k + 1,2 k−1√ 2 n < 2 k−1√ 2 + 1 √2.[2 k−1√ 2, 2 k−1√ )2 + √ 12sadrži ceo broj ako i samo ako je{2 k−1√ 2} > 1 − 1 √2,gde {x} označava razlomljeni deo realnog broja x. Pretpostavimo sada, suprotnotraženom tvr - denju, da navedeni intervali sadrže samo konačno mnogo celih brojeva.Tada postoji k 0 ∈ N tako da za sve k k 0 važiDefinišimo, za i 0, niz realnih brojeva:{2 k√ 2} 1 − 1 √2≈ 0, 292 . . . < 1 2 .r i = {2 i+k 0√2}.Očito, važi:r i+1 = {2r i } ={2ri r i < 1 2 ,2r i − 1 r i 1 2 .Po pretpostavci, imamo r i < 1 2 za sve i 0, pa je r i+1 = 2r i , odakle je r i = r 0 2 i(i 0). Sada imamor 0 2 i < 1 2za sve i 0, što je moguće samo ako je r 0 = 0. Me - dutim, r 0 = {2 k 0√2} jeiracionalan broj. Kontradikcija.71


42. Ma - darska, 1969. (konkurs ”Kürschák József”)Neka je n ceo broj. Dokazati: ako je brojprirodan, onda je on potpun kvadrat.Rešenje. Pretpostavimo da je2 + 2 √ 28n 2 + 1 = 22 + 2 √ 28n 2 + 1(1 + √ )28n 2 + 1 = mza neki prirodan broj m. Tada je √ 28n 2 + 1 neparan prirodan broj, odakle sledida m mora biti deljiv sa 4, tj. m = 4k za neko k ∈ N. Uvrštavajući ovo u gornjujednakost, dobijamo √ 28n 2 + 1 = 2k − 1, odakle kvadriranjem sledi 28n 2 + 1 =4k 2 − 4k + 1 = 4k(k − 1) + 1, odnosno7n 2 = k(k − 1).Budući da je (k, k − 1) = 1, postoje celi brojevi q, r tako da važi jedna od dvemogućnosti:(1) k = q 2 , k − 1 = 7r 2 ,(2) k = 7q 2 , k − 1 = r 2 .U slučaju (1) traženi zaključak neposredno sledi, pošto je tadam = 4k = 4q 2 = (2q) 2 .S druge strane, slučaj (2) je nemoguć, pošto bismo tada imali r 2 = 7q 2 − 1, štobi značilo da važi r 2 ≡ −1(mod 7). Me - dutim, lako se proverava da kvadrati celihbrojeva daju ostatke 0, 1, 2, 4 pri deljenju sa 7.Komentar. Može se pokazati da postoji beskonačno mnogo celih brojeva n za kojeje √ 28n 2 + 1 prirodan broj. Kao što se vidi iz gornjeg rešenja, ako je (q, r) rešenjePelove jednačinex 2 − 7y 2 = 1,tada n = qr ima traženu osobinu. Postojanje beskonačno mnogo rešenja ovejednačine pokazuje se potpuno analogno kao i u rešenju zadatka br. 40, posmatranjembinomnih razvoja (8 ± 3 √ 7) m (pri čemu uzimamo baš ove koeficijente jerje 8 2 − 7 · 3 2 = 64 − 63 = 1).72


43. MMO 1989. (Švedska)Za koje prirodne brojeve n postoji prirodan broj m tako da ni jedan od brojevam + 1, m + 2, . . . , m + n nije stepen prostog broja?Rešenje. Broj x nije stepen prostog broja ako i samo ako ima bar dva prosta faktora,što je ekvivalentno rastavljanju x = ab na dva uzajamno prosta činioca a, b > 1.Prema tome, ako želimo da obezbedimo da broj m + k ne bude stepen prostogbroja, možemo pokušati da posmatramo broj m deljiv sa k, jer tada iz m = km ′sledi m + k = k(m ′ + 1). Da bi k i m ′ + 1 bili uzajamno prosti, dovoljno je dak | m ′ .Pri svemu tome, mora biti k 2. Kako bismo obezbedili da ovaj uslov budeispunjen, uvedimo smenu m = m 0 + 1 i posmatrajmo niz brojevam 0 + 2, m 0 + 3, . . . , m 0 + n + 1.Kao što smo videli, uslov k | m 0 omogućava faktorizaciju m 0 + k = k(m ′ 0 + 1) zapogodno m ′ 0 ∈ N. Zbog toga, neka m 0 ima oblik m 0 = (n + 1)!m 1 . Tada za sve2 k n + 1 važi( )(n + 1)!m 0 + k = k m 1 + 1 .kU skladu sa gornjim razmatranjima, uslov k | m 1 povlači da()(n + 1)!k, m 1 + 1 = 1,kpa tada m 0 + k nije stepen prostog broja. Dakle, ako odaberemom 1 = (n + 1)!,imaćemo da nijedan od brojeva m 0 + 2, . . . , m 0 + n + 1 nije stepen prostog broja.Stoga traženi broj m postoji za sve n ∈ N, pošto smo upravo pokazali da je dovoljnouzetim = [(n + 1)!] 2 + 1.73


44. Predlog za MMO 1991. (Bugarska)Naći najveći prirodan broj k za koji1991 k ∣ ∣∣(1990 19911992 + 1992 19911990) .Rešenje. Radi bolje preglednosti, označimo a = 1991. Naš cilj je sada odre - divanjenajvećeg prirodnog broja k za kojia k ∣ ∣∣((a − 1) aa+1 + (a + 1) aa−1) .Pošto je a neparan broj, očito je da prvi sabirak u zagradi daje ostatak −1 prideljenju sa a, dok drugi daje ostatak 1. Zbog toga ćemo posebno razmotriti najvišestepene kojim a deli (a+1) aa−1 −1, odnosno (a−1) aa+1 +1 (ideja je sledeća: akose ispostavi da su ova dva najviša stepena različita, onda je manji od njih upravotraženo k).Razvijanjem izraza (a + 1) an po binomnom obrascu uočavamo da je poslednjisabirak 1, pretposlednji ( )a n 1 a = a n+1 , dok su ostali deljivi sa a n+2 . Ovo zapažanjesada potvrdujemo - indukcijom: dokazujemo da za sve prirodne brojeve n postoji s ntakav da a ∤ s n i(1 + a) an = 1 + s n a n+1 .Zaista, za n = 1 imamo:(1 + a) a = 1 +( ( ( a a aa + a1)2)2 + . . . + aa)a =( ( )a a= 1 + a(1 2 + + a + . . . = 1 + s 1 a2)3)2 .Pošto je a neparno, to a | ( a2), pa zato a ne deli( ( a as 1 = 1 + + a + . . .2)3)Iz pretpostavke da navedeno tvrdenje - važi za n, dobijamo:(1 + a) an+1 = ( 1 + s n a n+1) ( ) (a a a = 1 + s n a n+1 + s12)2 na 2n+2 + . . . =( )a= 1 + a(s n+2 n + s2)2 na n + . . . = 1 + s n+1 a n+2i pošto a ∤ s n , to a ∤ s n+1 .74


Na potpuno analogan način se pokazuje da za svaki prirodan broj n postojiprirodan broj t n takav da a ∤ t n i da važi(a − 1) an = −1 + t n a n+1 .Prema tome, iz gornja dva tvr - denja imamo da je a a najveći stepen od a kojideli (a + 1) aa−1 − 1 (prvo od gornja dva tvr - denja za n = a − 1), kao i da je najvećistepen kojim a deli (a − 1) aa+1 + 1 jednak a a+2 (drugo tvr - denje za n = a + 1).Otuda je traženi broj k max = a = 1991.75


45. Predlog za MMO 1982. (Australija)(a) Dokazati da (n + 1) ∣ ( )2nn za sve n ∈ N.(b) Za sve k ∈ N, naći najmanji prirodan broj C k takav da( )(n + k + 1)2n∣ C kn + kza sve n k.Rešenje. (a) Važi:)1 2n=n + 1( (2n)!n (n + 1)!n! = 1 ( ) 2n,n n − 1odakle dobijamo( ) ( )2n 2nn = (n + 1) .n n − 1Kako je (n, n + 1) = 1, to sledi (n + 1) | ( )2nn .(b) Iz prethodne tačke sledi da je C 0 = 1, pa zato pretpostavimo da je k > 0.Ako je n = k, tada je ( 2nn+k)= 1, što znači da mora biti Ck 2k + 1. Pokušaćemoda pokažemo da je baš C k = 2k + 1, tj. da važi( )(n + k + 1)2n∣ (2k + 1) n + kza sve n k. Kako je( ) 2n(2k + 1)n + k( ) 2n= [(n + k + 1) − (n − k)] ,n + kdovoljno je pokazati da( )(n + k + 1)2n∣ (n − k) .n + kMedutim, - na analogan način kao i u tački (a), dobijamo:( )( )2n(2n)!(n − k) =n + k (n + k)!(n − k − 1)! = (n + k + 1) 2n,n − k − 1odakle sledi željeni zaključak.76


46. Predlog za BMO, 1994. (Rumunija); Bugarska, 1994.Neka je p > 3 prost broj. Pokazati da je broj( ( ( )p p p+ + . . . + ⌊ ⌋1)2)2p3deljiv sa p 2 .Rešenje. Označimo sa S datu sumu binomnih koeficijenata i q =k q imamo: ( p p(p − 1)(p − 2) . . . (p − k + 1)= =k)k!=p · [(p − 1)(p − 2) . . . (p − k + 1)] · [(k + 1) . . . q]q!⌊2p3.⌋. Za 1 Naravno, pošto je q < p, sledi da p ∤ q!. Takode, - svaki od binomnih koeficijenatakoji čine S je očito deljiv sa p, pa je tvrdenje - koje dokazujemo, p 2 | S, ekvivalentnosa p | q!pS. Pri tome je iz gornje jednakosti( )q! p= [(p − 1)(p − 2) . . . (p − k + 1)] · [(k + 1) . . . q].p kZa izraz u prvoj od dve uglaste zagrade važiodakle je( )q! pp kPrema tome,(p − 1)(p − 2) . . . (p − k + 1) ≡ (−1) k−1 (k − 1)! (mod p),≡ (−1) k−1 k−1 q!(k − 1)!(k + 1) . . . q = (−1) (mod p).kq!q∑p S ≡ q!k=1(−1) k−1(mod p).kZbog toga, zadatak će biti rešen ako pokažemo da je brojilac sledećeg racionalnogbroja (nakon skraćivanja) deljiv sa p:q∑ (−1) k.kk=177


Pošto je p > 3, prost broj p mora biti oblika 6m ± 1. Ako je p = 6m + 1, tadaimamo q = 4m i−1 + 1 2 − 1 3 + . . . + 14m =[− 1 + 1 2 + . . . + 1 ( 14m − 2 2 + 1 4 + . . . + 1 )]=4m(= − 1 + 1 2 + . . . + 14m − 1 − 1 2 − . . . − 1 )=2m( 1= −( 1= −2m + 1 + 12m + 2 + . . . + 14m)=2m + 1 + 14m + 12m + 2 + 14m − 1 + . . . )=( 6m + 1= −4m(2m + 1) + 6m + 1(= −p(4m − 1)(2m + 2) + . . . + 6m + 1 )=3m(3m + 1))14m(2m + 1) + 1(4m − 1)(2m + 1) + . . . + 1=3m(3m + 1)pA= −2m(2m + 1) . . . 4m= −pA(2m− 1)!q!za neki prirodan broj A. Pošto p ∤ q!, posmatrani razlomak će nakon skraćivanjaimati brojilac deljiv sa p.Slučaj p = 6m − 1 se razmatra analogno.78


47. MMT, Mersch, Luksemburg, 1980. (Jugoslavija)Neka je p prost broj, a n prirodan broj. Dokazati da su sledeća dva tvr - denja ekvivalentna:(i) Nijedan od binomnih koeficijenata ( nk), 1 k < n, nije deljiv sa p.(ii) n = p s q − 1 za neke cele brojeve s, q takve da je s 0 i 0 < q < p.Rešenje. (i) ⇒ (ii) Pošto polazimo od pretpostavke da p ne deli( n n(n − 1) . . . (n − k + 1)=k)1 · 2 · . . . · kza sve 1 k n − 1, imamo da ta pretpostavka važi specijalno za sve k < p (semako je n < p − 1, no tada je trivijalno n = p s q − 1 za s = 0 i q = n + 1, pa jetražena implikacija dokazana), pa i za k = p − 1. Pošto p ∤ (p − 1)!, sledi da brojn(n − 1) . . . (n − p + 2)nije deljiv sa p, što znači da n ne može davati nijedan od ostataka 0, 1, . . . , p − 2pri deljenju sa p. Drugim rečima, n ≡ −1(mod p), tj. p | (n + 1). Stoga za nekes, q > 0 važi n = p s q − 1, pri čemu p ∤ q. Preostaje da se pokaže da mora bitiq < p.Me - du ”uzastopnim” binomnim koeficijentima imamo sledeću vezu:( n=k)n . . . (n − k + 2)(n − k + 1)k!= n + 1 − k ( ) n. (23)k k − 1S obzirom na to, brojilac i imenilac razlomka n+1−kkmoraju za sve 1 k nsadržati prost faktor p sa istim stepenom, jer bismo u suprotnom za najmanje kkoje nema tu osobinu imali da je ( nk)deljivo sa p. Medutim,-n + 1 − kk= ps q − kkpa odmah imamo da važi jednakost najviših stepeni kojima p deli brojilac i imenilacposmatranog razlomka kad god p s ∤ k. Prema tome, dovoljno je posmatrati brojeveoblika k = p s m, m q. Tada je,n + 1 − kk= ps q − p s mp s m79= q − mm .


Ukoliko bi bilo q > p (slučaj q = p je očito nemoguć, jer p ∤ q), tada bismoza m = q − p imali kontradikciju sa (i): brojilac posmatranog razlomka bi bioq − (q − p) = p, a imenilac q − p, pri čemu p ∤ (q − p). Stoga zaključujemo damora biti q < p, što okončava dokaz tražene implikacije.(ii) ⇒ (i) Ova implikacija se neposredno dokazuje indukcijom, koristeći jednakost(23). Naime, za k = 1, ( n1)= n očito nije deljivo sa p. Ukoliko podemo-od pretpostavke da p ∤ ( nk−1), tada je na osnovu (23) za kompletiranje induktivnogkoraka dovoljno dokazati da su najviši stepeni kojima p deli n + 1 − k i k jednaki.Neka je k = p r m, gde p ∤ m i r 0. Ako je r < s, tada jen + 1 − k = p s q − p r m = p r (p s−r q − m)deljivo sa p r , ali ne i sa p r+1 (baš kao i k). S druge strane, ako je r = s (slučajr > s je jasno nemoguć, jer bismo tada imali k p s+1 > p s q > n), tada jen + 1 − k = p s (q − m),i pri tome p ∤ (q − m), jer je m < q < p. Zato su i n + 1 − k i k deljivi sa p s , ali nei sa p s+1 . Time je induktivni korak dokazan, pa odmah sledi tražena implikacija.80


48. Čehoslovačka, 1988.Naći sve prirodne brojeve m tako da važi:( )m(m + 1)1! 3! 5! . . . (2m − 1)! =! .2Rešenje. Ideja rešenja je da se posmatraju prosti faktori leve i desne strane datejednačine. Pošto za proizvoljan prost broj p i prirodan broj n važi da p ∤ n! ako isamo ako n < p, zaključujemo da egzistencija prostog broja p takvog da2m − 1 < p m(m + 1)2povlači da m ne može biti rešenje (jer je tada desna strana deljiva sa p, dok levanije). Po Bertranovom stavu, takav prost broj će postojati kad god jem(m + 1)2 2(2m − 1),što je ekvivalentno sa m 2 − 7m + 4 0. Medutim, - ova nejednakost je očito tačnaza m 7, pa preostaju da se razmotre brojevi m 6.Za m = 6 je 2m − 1 = 11 < 13 < 21 = 6·72, dok za m = 5 imamo2m − 1 = 9 < 11 < 15 = 5·62, pa ove vrednosti m nisu rešenja posmatranejednačine.S druge strane, m = 1, 2 očigledno jesu rešenja. Rešenje je i m = 3, jer je1! 3! 5! = 6 · 5! = 6!, a takode - i m = 4, pošto je 1! 3! 5! 7! = 6! 7! = 720 · 7! =10 · 9 · 8 · 7! = 10!.81


49. Predlog za MMO, 1989. (Kolumbija)Neka f(m) označava najveći ceo broj k za koji 2 k | m!. Dokazati da za sveprirodne brojeve n postoji beskonačno mnogo prirodnih brojeva m tako da jem − f(m) = n.Rešenje. Po Ležandrovoj formuli,f(m) = ∑ ⌊ m⌋2 k .k1Ako je a r a r−1 . . . a 1 a 0 binarni prikaz broja m, tj.m = a r 2 r + . . . + a 1 2 1 + a 0 ,tada za 1 k r važi⌊ m2 k ⌋= a r 2 r−k + . . . + a k+1 2 1 + a k = a r . . . a k+1 a k(za k > r posmatrani ceo deo je 0). Dakle,r∑ r∑f(m) = a i 2 i−k =k=1 i=kr∑a ii=1 k=1i∑2 i−k =r∑a i (2 i − 1).i=1Zato jem − f(m) = a r + . . . + a 1 + a 0 .Prema tome, m − f(m) = n ako i samo ako m u binarnom zapisu ima tačno njedinica. Takvih brojeva ima očito beskonačno mnogo.82


50. Predlog za MMO, 1985. (Bugarska)Neka su a i b celi, a n prirodan broj. Dokazati:()n−1∏n!b n−1 (a + kb) .∣k=0Rešenje. Ako je p prost broj i s 0 najveći ceo broj za koji p s | n!, tada je poLežandrovoj teoremi:s = ∑ m1⌊ np m ⌋∞∑m=1⌊ n2 m ⌋< n, (24)pa je s n − 1 (isto pomoćno tvr - denje smo imali u zadatku br. 35). Ako p | b,tada p n−1 | b n−1 , a time i p s | b n−1 . U suprotnom, p ∤ b, pa p deli bar jedanod a, a + b, . . . , a + (p − 1)b, pošto ovaj niz od p brojeva tada čini potpun sistemostataka po modulu p (u suprotnom bi dva takva broja davala isti ostatak (mod p),pa bi p | (i − j)b za neke i, j ∈ {0, 1, . . . , p − 1}, što je nemoguće). Zato p delibar ⌊n/p⌋ faktora proizvoda, datog u zadatku, p 2 deli bar ⌊ n/p 2⌋ tih faktora, itd.Zaključak je da je stepen prostog broja p u tom proizvodu bar s (vidi (24)), odaklesledi tvr - denje zadatka.83


51. Predlog za MMO, 1983. i 1991. (S.S.S.R.)Neka je a n poslednja nenula cifra u dekadnom prikazu broja n!. Da li niz a 1 , a 2 , . . .može biti, počev od nekog člana, periodičan?Rešenje. Ideja rešenja je da posmatramo jednake uzastopne članove datog nizacifara, pa da na osnovu dobijenih informacija pokušamo da dokažemo negativanodgovor na postavljeno pitanje. Naime, ako definišemo skupA = {n : a n−1 = a n },tada iz pretpostavke da je dati niz periodičan za n n 0 sa periodom T dobijamoda iz n n 0 i n ∈ A (tj. a n−1 = a n ) sledia n−1+kT = a n+kT ,odnosno n + kT ∈ A, za sve k 0. Koristeći osobine skupa A, dokazaćemo daovo nije moguće.Pokušaćemo najpre da iz uslova a n−1 = a n saznamo nešto o samom broju n.Prikažimo (n−1)! = 10 q (10u+a n−1 ) i n = 10 r (10v+x), pri čemu je 1 x 9.Tada jen! = (n − 1)! · n = 10 q+r (10s + a n−1 x),gde je s = 10uv + ux + a n−1 v.Ukoliko 5 ∤ a n−1 x, tada je a n , poslednja nenula cifra od n!, jednaka cifri jedinicaproizvoda a n−1 x. Ta cifra treba da bude jednaka a n−1 , tj. važi10 | (a n−1 x − a n−1 ) = a n−1 (x − 1).Pošto ispitujemo slučaj kada 5 ∤ a n−1 x, sledi da 5 | (x − 1), odnosno x ∈ {1, 6}.Drugi slučaj je kada 5 | a n−1 x, tj. a n−1 = 5 ili x = 5. Ispitaćemo da li mogućajednakost a t = 5 za neko t. U tu svrhu, prikažimo t! = 2 α 5 β γ, gde γ nije deljivoni sa 2, ni sa 5. Po Ležandrovoj formuli je⌊ ⌊ ⌊ ⌊ ⌋t t t tα = + + + . . . +2⌋4⌋8⌋2 i + . . . ,β =⌊ ⌊ ⌋ ⌊ ⌋ ⌊ ⌋t t tt+5⌋5 2 +5 3 + . . . +5 i + . . . ,pa se lako pokazuje da važi α > β kad god je t 2. Zbog toga je t! = 10 β (2 α−β γ),što znači da se a t poklapa sa cifrom jedinica broja 2 α−β γ. Za t 2, ta cifra je84


očigledno parna, dok je a 1 = 1. Prema tome, a t ≠ 5 za sve t ∈ N, pa iz 5 | a n−1 xsledi x = 5.Sumirajući prethodna razmatranja, možemo zaključiti da n ∈ A povlači da jeposlednja nenula cifra broja n jedna od 1, 5, 6.Uočimo sada da za proizvoljno m ∈ N imamo (10 m )! = 10 m · (10 m − 1)!,zbog čega je 10 m ∈ A. Kao što je ranije rečeno, tada za sve k 0 i za sve m takveda je 10 m n 0 mora biti10 m + kT ∈ A.Neka jeT = 2 a 5 b c,pri čemu je c neparan broj koji nije deljiv sa 5. Ideja je da pokažemo da T imaumnožak lT čija poslednja nenula cifra nije nijedna od 1, 5, 6, jer tada za dovoljnoveliko m (naime, za m takvo da je 10 m > max(lT, n 0 )) važi da je poslednja nenulacifra broja 10 m + lT ista kao i poslednja nenula cifra broja lT . Pošto ta cifra nijeu skupu {1, 5, 6}, dobićemo kontradikciju sa uslovom 10 m + lT ∈ A (koji sledi izprepostavljene periodičnosti niza cifara a n ).Me - dutim, traženi umnožak se lako nalazi. Naime, imamo2 b 5 a T = 10 a+b c,pa je poslednja nenula cifra broja 2 b 5 a T ista kao i cifra jedinica broja c. Ako ta cifranije 1 (već 3, 7 ili 9), željeni cilj je postignut. Ali, ako je ta cifra 1, posmatrajmodvostruko veći broj2 b+1 5 a T = 10 a+b · 2c.Poslednja nenula cifra ovog broja je cifra jedinica broja 2c, a to je u posmatranomslučaju 2. Time je dokaz neperiodičnosti niza a n kompletiran.85


52. MMO, 1978. (Kuba)Posmatrajmo sve parove prirodnih brojeva (m, n), m < n, sa osobinom da seposlednje tri cifre u dekadnom zapisu brojeva 1978 m i 1978 n poklapaju. Naći svetakve parove (m, n) za koje je m + n minimalno.Rešenje. Uslov zadatka možemo zapisati kao1978 m (1978 n−m − 1) = 1000q = 2 3 5 3 q,za neko q 1, odakle dobijamo 8 | 1978 m i 125 | (1978 n−m − 1). Kako je1978 = 2 · 989, to prvi uslov daje m 3. Iz drugog uslova imamo:1 ≡ 1978 n−m ≡ (−2) n−m (mod 5),što je moguće samo ako je n − m = 4k za neki prirodan broj k. Preostaje daodredimo najmanji prirodan broj k za koji je 1978 4k −1 deljivo sa 125. Neposrednodobijamo1978 4 ≡ 6(mod 125),pa se posmatrani uslov svodi naIz Ojlerove teoreme sledi6 k ≡ 1(mod 125).6 100 = 6 ϕ(125) ≡ 1(mod 125).Kako je 6 100 − 1 = (6 50 − 1)(6 50 + 1) i poslednja cifra broja 6 50 + 1 je 7, sledida 125 | (6 50 − 1). Ponavljajući ovaj argument još jednom u odnosu na 6 50 − 1 =(6 25 − 1)(6 25 + 1), dobijamo da 125 | (6 25 − 1). Zbog toga, najmanje k za koje125 | (6 k − 1) zadovoljava k | 25, tj. k ∈ {1, 5, 25}. Direktno proveravamo da6 5 −1 nije deljivo sa 125 (kao ni 6 1 −1). Stoga je k = 25 najmanji broj sa traženomosobinom, pa je suma m + n minimalna za n = 103, m = 3.86


53. BMO, 1984. (Bugarska)Dokazati da za sve prirodne brojeve m postoji prirodan broj n > m takav da sedekadni zapis broja 5 n dobija dopisivanjem izvesnog broja cifara sleva dekadnomzapisu broja 5 m .Rešenje. Uslov zadatka se može zapisati kao 10 r | (5 n − 5 m ), gde je r broj cifarau dekadnom zapisu broja 5 m , tj. r = ⌊log 10 5 m ⌋ + 1. Pošto je r m, posmatranarelacija deljivosti je ekvivalentna sa 2 r | (5 n − 5 m ) = 5 m (5 n−m − 1), tj. saPo Ojlerovoj teoremi, važi:Ali, tada je očito da se za n oblika2 r | (5 n−m − 1).5 ϕ(2r) ≡ 1(mod 2 r ).n = m + ϕ(2 r )k = m + 2 r−1 k,k ∈ N, dobijašto se i tražilo.5 n = 5 m (5 ϕ(2r) ) k ≡ 5 m (mod 2 r ),87


54. Predlog za MMO, 1976. (Vijetnam)Dokazati da postoji beskonačno mnogo prirodnih brojeva n takvih da se u dekadnomzapisu broja 5 n pojavljuje 1976 uzastopnih nula.Rešenje. Zadatak će biti rešen ako dokažemo da postoji beskonačno mnogo prirodnihbrojeva n takvih da se 5 n može zapisati u obliku5 n = 10 k+1976 q + r,za neke prirodne brojeve q, r, k, pri čemu je r < 10 k . Da bi to bilo ispunjeno,mora biti 10 k+1976 < 5 n , odakle je n > k + 1976, pa zato 5 k+1976 | r. Pišimor = 5 k+1976 s, pri čemu će s biti naknadno odre - deno. Zbog 5 k+1976 | r < 10 k ,sledi 5 k+1976 < 10 k , tj. 2 k > 5 1976 . To znači da mora bitik ⌊1976 log 2 5⌋ + 1 = 4589.Sada posmatrana jednačina glasi 5 n−k−1976 = 2 k+1976 q+s, odnosno, nakon smenet = k + 1976:5 n−t = 2 t q + s,gde je q > 0 (što je ekvivalentno sa n − t > 0). Kako bismo mogli da primenimoOjlerovu teoremu, zgodno je da odaberemo s = 1. Pošto je5 ϕ(2t) ≡ 1(mod 2 t ),svi traženi uslovi su ispunjeni za sve t 4589 + 1976 = 6565 i sve n za koje važin > t iϕ(2 t ) = 2 t−1 | (n − t).Drugim rečima, za sve m ∈ N i t 6565, brojima željenu osobinu.n = 2 t−1 m + t88


55. Predlog za BMO, 1989. (Kipar)Dokazati da1989∣(n nnn − n nn)za sve prirodne brojeve n 3.Rešenje. Radi kraće notacije, označimo broj u zagradi sa a. Ovaj broj se možefaktorisati kaoa = n nnn − n nn = n nn (n nnn −n n − 1),što motiviše da označimoNajzad, neka je c = n n − n. Sada jeb = n nn − n n = n n (n nn −n − 1).a = n nn (n b − 1), b = n n (n c − 1), c = n(n n−1 − 1).Kako je 1989 = 9 · 13 · 17, ideja rešenja je da dokažemo da ϕ(d) | b za sved ∈ {9, 13, 17}, budući da tada važi:(n, d) = 1 ⇒ d | (n ϕ(d) − 1) | (n b − 1) | a.S druge strane, ako za d ∈ {13, 17} imamo (n, d) ≠ 1, tada d | n | a, dok iz(n, 9) ≠ 1 sledi 3 | n, pa 9 | n 2 | a. Prema tome, zadatak će biti rešen akopokažemo da [ϕ(9), ϕ(13), ϕ(17)] = [6, 12, 18] = 48 | b.Najpre, 3 | n ⇒ 3 | b. S druge strane, ako n nije deljivo sa 3, imamo (n, 3) = 1.No, tada 2 | c (jer to važi ako je n parno, a u suprotnom je n n−1 − 1 parno). Dakle,ϕ(3) = 2 | c, pa po Ojlerovoj teoremi 3 | (n c − 1) | b.Dalje, ako je n parno, tada 2 n | b, znači 16 | b, pošto je n 4. U suprotnom,neka je n = 2k + 1. Sada imamo:c = (2k + 1)((2k + 1) 2k − 1) = (2k + 1)((4k(k + 1) + 1) k − 1) =k(k + 1)= (2k + 1)(4k(k + 1))N = 8(2k + 1) N.2Prema tome, ϕ(16) = 8 | c i zato, ponovo se koristeći Ojlerovom teoremom, sledi16 | (n c − 1) | b. Time je rešenje kompletirano.89


56. Predlog za MMO, 1984. (Kanada)(a) Dokazati da postoji beskonačno mnogo parova prirodnih brojeva (m, n) takvihda je 4mn − m − n + 1 potpun kvadrat.(b) Dokazati da ne postoji nijedan par prirodnih brojeva (m, n) takav da je 4mn −m − n potpun kvadrat.Rešenje. (a) Posmatramo jednačinu4mn − m − n + 1 = k 2 .Nakon množenja sa 4 i oduzimanja 3 od obe strane, moguće je faktorisati levustranu, tako da se dobija(4m − 1)(4n − 1) = 4k 2 − 3.Ideja koja vodi nalaženju beskonačnog niza rešenja ove jednačine je da se za kuvede odgovarajuća smena koja će 4k 2 − 3 transformisati u razliku kvadrata. Očigledno,linearna smena oblika k = at + b (a, b ∈ Z) nije odgovarajuća, jer nakonkvadriranja ostaje linearni član. Zbog toga ćemo pokušati sa smenom k = at 2 + b.Tada je4k 2 − 3 = 4(at 2 + b) 2 − 3 = 4a 2 t 4 + 8abt 2 + (4b 2 − 3) == (2at 2 + b) 2 − (−4abt 2 ) + (3b 2 − 3).Prema tome, ako izaberemo koeficijente a, b tako da je b 2 = 1, ab < 0 i da −abbude potpun kvadrat, postići ćemo naš cilj. Očito je a = 1, b = −1 jedan odadekvatnih izbora, pa tako za k = t 2 − 1 imamo4k 2 − 3 = (2t 2 − 1) 2 − 4t 2 = (2t 2 − 2t − 1)(2t 2 + 2t − 1).Kako su brojevi 2t 2 − 2t = 2t(t − 1) i 2t 2 + 2t = 2t(t + 1) deljivi sa 4, možemonjih ”proglasiti” za 4m, odnosno 4n. Tako, imamo željeni niz rešenja: naime, akoje t ∈ N, zam = 1 2 t(t − 1), n = 1 t(t + 1),2važi 4mn − m − n + 1 = (t 2 − 1) 2 .(b) Slično kao malopre, jednačina 4mn − m − n = k 2 je ekvivalentna sa(4m − 1)(4n − 1) = 4k 2 + 1.90


Pretpostavimo, suprotno tvr - denju zadatka, da postoje prirodni brojevi m, n, k zakoje važi ova jednakost. Neka je p prost broj koji deli 4m − 1. Tada je(2k) 2 ≡ −1(mod p).S druge strane, po maloj Fermaovoj teoremi jeKako je p očigleno neparan, sledi(2k) p−1 ≡ 1(mod p).1 ≡ (2k) p−1 = ((2k) 2 ) p−12 ≡ (−1) p−12 (mod p),odakle je p ≡ 1(mod 4). Kako je u ovom razmatranju prost faktor p od 4m − 1 bioproizvoljan, zaključujemo (nakon što napišemo 4m−1 kao proizvod svojih prostihfaktora) da je4m − 1 ≡ 1(mod 4),kontradikcija.91


57. (a),(b): Rumunija, 1978.; (c): Predlog za MMO, 1985. (Rumunija)(a) Dokazati da za svaki prirodan broj a 3 postoji beskonačno mnogo prirodnihbrojeva n za koje n | (a n − 1).(b) Naći sve prirodne brojeve n za koje n | (2 n − 1).(c) Neka je k 2 i neka su n 1 , n 2 , . . . , n k prirodni brojevi takvi dan i+1 |(2 n i− 1)za sve 1 i k − 1, kao i n 1 |(2 n k− 1). Dokazati da je n 1 = . . . = n k = 1.Rešenje. (a) Pretpostavimo da prirodan broj n deli m = a n − 1. Tada je m = nkza neko k ∈ N, pa jea m − 1 = (a n ) k − 1 = (a n − 1)(a n(k−1) + . . . + a n + 1).Zaključujemo da tada m | (a m − 1). Stoga, ako definišemo nizn 1 = 1, n r+1 = a nr − 1, (r 1)tada iz prethodnih razmatranja neposredno sledi da svaki član ovog niza ima željenuosobinu. Preostaje da uočimo da iz pretpostavke a 3 sledi da je ovaj niz strogorastući, tj. da se sastoji iz različitih brojeva.(b) Očigledno, n = 1 zadovoljava traženi uslov. Pretpostavimo da postoji n 2 koje zadovoljava navedeni uslov. Neka q n označava najmanji prost faktor brojan. Dokažimo da važi: ako je n > 1 i p | (2 n − 1), tada je p > q n . U tom slučaju,imaćemo očiglednu kontradikciju, jer n | (2 n − 1) povlači q n | (2 n − 1).Najpre, treba primetiti da ako za prirodne brojeve a, b važi2 a ≡ 2 b ≡ 1(mod p),tada je2 (a,b) ≡ 1(mod p).Naime, ako je a b i a = qb + r, tada važi2 r ≡ (2 b ) q 2 r = 2 a ≡ 1(mod p).Nastavljajući očiglednu primenu Euklidovog algoritma u eksponentu, dobijamoupravo željeni zaključak. Sada, pošto po maloj Fermaovoj teoremi važi 2 p−1 ≡1(mod p), za d = (n, p − 1) imamo 2 d ≡ 1(mod p). Zbog toga je d > 1, pa važiq n d. S druge strane, d | (p − 1), što implicira p > d q n .Prema tome, n = 1 je jedino rešenje.92


(c) Pretpostavimo da je n 1 > 1. Tada je redom n k > 1,. . . , n 2 > 1. Neka q noznačava, kao i u (b), najmanji prost faktor broja n. Iz uslova zadatka slediq n2 | 2 n 1− 1,pa po pomoćnom tvr - denju iz prethodnog zadatka (p | (2 n − 1) ⇒ q n < p) slediq n1 < q n2 . Ponavljajući ovo razmatranje, dobijamo:Kontradikcija.q n1 < q n2 < . . . < q nk < q n1 .93


58. S.A.D., 1991.Dokazati da je za svaki prirodan broj n ∈ N, nizkonstantan počev od nekog člana.2, 2 2 , 2 22 , 2 222 , . . . (mod n)Rešenje. Označimo sa a i , i 1, članove datog niza (preciznije, ovaj niz je definisansa a 1 = 2 i a i+1 = 2 a iza i 1). Zadatak zapravo traži da se pokaže da je za sveprirodne brojeve n razlika a i+1 − a i deljiva sa n za dovoljno veliko i. Drugimrečima, ako označimo b i = a i+1 − a i , cilj je da se dokaže:za sve n ∈ N postoji i n ∈ N tako da i i n ⇒ n | b i . (25)Najpre ćemo malo transformisati b i . Naime, važi:b i = a i+1 − a i = 2 a i− 2 a i−1= 2 a i−1(2 a i−a i−1− 1) = a i (2 b i−1− 1),pri čemu smo po potrebi označili a 0 = 1. Koristeći ovu vezu, dokazaćemo tvr - denje(25) indukcijom po n. Ono je očigledno za n = 1, pa po - dimo od pretpostavke daje ono tačno za sve prirodne brojeve manje od neke odabrane vrednosti n.Razložimo n = 2 k q, gde je q neparan broj. Pošto za sve k a i važi 2 k | 2 a i=a i+1 , to iz činjenice da niz a i nije ograničen sledi da postoji j k tako da 2 k | a i | b iza sve i j k . S druge strane, želimo da q | b i za dovoljno veliko i, što je mogućesamo ukoliko q | (2 b i−1− 1). Po Ojlerovoj teoremi je2 ϕ(q) ≡ 1(mod q),pa bi iz ϕ(q) | b i−1 sledio željeni zaključak. Me - dutim, po induktivnoj pretpostavci(koju možemo da primenimo budući da je ϕ(q) < q n), ova deljivost je tačna zasve i za koje je i − 1 i ϕ(q) . Dakle, za sve i i ϕ(q) + 1 važi q | (2 b i−1− 1) | b i .Kako je (2 k , q) = 1, sada za sve i max(j k , i ϕ(q) + 1) imamo n = 2 k q | b i , štookončava induktivni dokaz.94


59. MMO, 1990. (Rumunija)Naći sve prirodne brojeve n za koje n 2 | (2 n + 1).Rešenje. Kako je 2 n + 1 neparan broj, to n mora biti takode - neparan. Pišimon = 3 k d, gde je d neparan broj koji nije deljiv sa 3. Početna ideja je da analiziramostepen trojke koji deli broj 2 n + 1. Najpre, imamo faktorizaciju[]2 n + 1 = (2 3k ) d + 1 = (2 3k + 1) (2 3k ) d−1 + . . . + 2 3k + 1 .Budući da je (2 3k ) m ≡ (−1) m (mod 3), i kako u uglastoj zagradi imamo neparnomnogo sabiraka, broj u toj zagradi nije deljiv sa 3. Prema tome, stepen kojim 3 deli2 n + 1 je jednak stepenu kojim 3 deli 2 3k + 1.Izraz 2 3k + 1 možemo faktorisati uzastopnom primenom formule za zbir kubova:2 3k + 1 = (2 + 1)(2 2 − 2 1 + 1) . . . (2 2·3k−1 − 2 3k−1 + 1). (26)Sada želimo da ustanovimo stepen trojke u broju oblika 2 2q − 2 q + 1, gde je qneparan broj. Lako se vidi da je on deljiv sa 3, pa ćemo posmatrati njegov ostatakpri deljenju sa 9. Pošto je 2 6 ≡ 1(mod 9), pisaćemo q u obliku q = 6s + r,r ∈ {1, 3, 5}, jer je tada2 q = (2 6 ) s 2 r ≡ 2 r (mod 9).Stoga je 2 2q − 2 q + 1 ≡ 2 2r − 2 r + 1(mod 9), pa se direktnom proverom zaključujeda je uvek2 2q − 2 q + 1 ≡ 3(mod 9).Dakle, svaki od brojeva u zagradama na desnoj strani razlaganja (26) je deljiv sa3, ali ne i sa 9. Otuda je 2 3k + 1 (a time i 2 n + 1) deljiv sa 3 k+1 , ali ne i sa 3 k+2 .Ali, po uslovu zadatka, 2 n + 1 treba da je deljivo sa n 2 , pa tako i sa 3 2k . Zato je2k k + 1, tj. k ∈ {0, 1}.Sada prelazimo na razmatranje broja d. Ako je d = 1, tada je n ∈ {1, 3} i lakouočavamo da oba ova broja predstavljaju rešenje zadatka. Zbog toga, pretpostavimoda je d > 1. Neka je p najmanji prost faktor broja d. Jasno, mora biti p 5.Tako - de, imamo da je2 n ≡ −1(mod p), (27)pa važi 2 2n ≡ 1(mod p). S druge strane, po maloj Fermaovoj teoremi je 2 p−1 ≡1(mod p), odakle na potpuno analogan način kao u zadatku br. 57 (b) dobijamo2 l ≡ 1(mod p), gde je l = (2n, p − 1). Me - dutim, svi prosti faktori broja p − 1 suočito manji od p, najmanjeg prostog faktora od d, pa sledi (d, p − 1) = 1. Otuda je95


l = (2 · 3 k , p − 1), gde je k ∈ {0, 1}, tj. l ∈ {1, 2, 3, 6}. Pošto p | (2 l − 1), p delijedan od brojeva 1, 3, 7, 63. Zbog ograničenja p 5, jedina mogućnost je p = 7.Ali, lako se proverava da stepen dvojke pri deljenju sa 7 može davati isključivoostatke 1, 2, 4, što je kontradikcija sa (27).Prema tome, jedini brojevi sa traženom osobinom su n = 1 i n = 3.Komentar. Primetimo da je prvi deo gornjeg rešenja gotovo analogan idejama kojise pojavljuju u rešenju zadatka br. 2 (b), a kojeg je Rumunija predložila za MMOgodinu dana ranije. Može se sa velikom sigurnošću reći da ova dva zadatka potičuod istog autora.96


60. BMO, 1989. (Bugarska)Naći sve prirodne brojeve n za koje jed 2 1 + d 2 2 + d 2 3 + d 2 4 = n,gde su 1 = d 1 < d 2 < . . . < d k = n (k 4) svi pozitivni delioci broja n.Rešenje. Ako bi bilo d 2 > 2, tada bi n bio neparan broj, kao i svi njegovi deliteljid i , ali bi tada broj d 2 1 + d2 2 + d2 3 + d2 4 bio paran, što je kontradikcija. Zaključujemoda je d 2 = 2, pa je tačno jedan od brojeva d 3 , d 4 paran.Ako bi d 3 bio paran, d 3 = 2a, sledilo bi a | n i 1 < a < d 3 , pa je to mogućesamo ako je a = 2, tj. d 3 = 4. Tada je n = 4m, pa je 4m = 1 2 + 2 2 + 4 2 + d 2 4 ,odnosno d 2 4 ≡ −1(mod 4), što je nemoguće.Prema tome, d 3 je neparan broj, a d 4 je paran, d 4 = 2b, pri čemu je b > 1.Kako tada b | n, to je b = 2 ili b = d 3 . Medutim, - prvi slučaj je nemoguć, jer jetada d 4 = 4, odakle je d 3 = 3 i n = 1 2 + 2 2 + 3 2 + 4 2 = 30, što nije deljivo sa 4.Dakle, d 3 = b, pa važi n = 2bm za neki prirodan broj m. Dobijamo relaciju5(b 2 + 1) = 2bm,pa mora biti b = 5. Otuda je d 4 = 10, n = 130, što predstavlja jedino rešenjezadatka.97


61. S.S.S.R., 1990. (republička olimpijada)Rešiti u skupu celih brojeva:x 5 − x 3 − x 2 + 1 = y 2 .Rešenje. Faktorizacijom leve strane dobijamo(x − 1) 2 (x + 1)(x 2 + x + 1) = y 2 .Ako je x = 1, tada je y = 0. U slučaju x ≠ 1, gornju jednačinu transformišemo uoblik( ) y 2(x + 1)(x 2 + x + 1) = .x − 1Dakle, (x − 1) | y. Obeležimo A = x + 1, B = x 2 + x + 1. Budući da jeB − xA = 1, sledi (A, B) = 1. Kako je B = ( x + 2) 1 2 +34> 0, A i B supotpuni kvadrati. No, za x > 1 je x 2 < x 2 + x + 1 < (x + 1) 2 , a za x −2 jex 2 > x 2 + x + 1 > (x + 1) 2 , pa zato B ne može biti kvadrat celog broja. Preostajuslučajevi x = −1 i x = 0 iz kojih redom dobijamo y = 0, odnosno y = ±1.98


62. Ma - darska, 1990.Da li jednačinaima racionalna rešenja?x 2 + xy + y 2 = 2Rešenje. Podimo - od pretpostavke da data jednačina ima racionalno rešenje (x, y).Tada možemo pisati x = a c i y = b c, pri čemu ove razlomke ne možemo skratitiistim prirodnim brojem: drugim rečima, (a, b, c) = 1 (pri čemu je a, b, c ∈ Z).Dobijamo:a 2 + ab + b 2 = 2c 2 .Iz gornje jednakosti sledi da oba broja a, b moraju biti parna (u suprotnom bi levastrana sadržala jedan ili tri neparna sabirka). Zbog toga, c je neparan broj (u suprotnombi bilo 2 | (a, b, c)). Me - dutim, sada je leva strana gornje jednakosti deljiva sa4, dok je desna strana deljiva sa 2, ali ne i sa 4. Kontradikcija. Dakle, odgovor napostavljeno pitanje je negativan.Komentar. Isti rezultat sledi i za jednačinu x 2 + nxy + y 2 = 2, gde je n bilo kojineparan ceo broj.99


63. S.A.D., 1976.Naći sva celobrojna rešenja jednačinea 2 + b 2 + c 2 = a 2 b 2 .Rešenje. Očigledno, a = b = c = 0 jeste rešenje date jednačine. Stoga pretpostavimoda je bar jedan od brojeva a, b, c različit od 0 i definišimo d = (a, b, c). Tadaje a = a 1 d, b = b 1 d i c = c 1 d za neke a 1 , b 1 , c 1 ∈ Z takve da je (a 1 , b 1 , c 1 ) = 1.Uvrštavajući ovo u polaznu jednačinu, sledia 2 1 + b 2 1 + c 2 1 = a 2 1b 2 1d 2 .Desna strana (budući da je u pitanju potpun kvadrat) daje ostatak 0 ili 1 pri deljenjusa 4. Pri tome, taj ostatak može biti 1 samo ako su a 1 i b 1 (a i d) neparni brojevi.Medutim, - tada je ostatak koji a 2 1 + b2 1 + c2 1 daje pri deljenju sa 4 jednak 2 ili 3, paodmah dobijamo kontradikciju. S druge strane, ako je razmatrani ostatak 0, tada jea 2 1 + b 2 1 + c 2 1 ≡ 0(mod 4),što je moguće samo ako su a 1 , b 1 , c 1 parni. Ali, to je u suprotnosti sa pretpostavkom(a 1 , b 1 , c 1 ) = 1, pa je trivijalno ujedno i jedino rešenje posmatrane jednačine.100


64. Predlog za MMO, 1982. (Belgija)Naći sva celobrojna rešenja jednačinex 3 − y 3 = 2xy + 8.Rešenje. Posmatrajmo najpre trivijalne slučajeve: za x = 0 očito dobijamo y =−2, dok za y = 0 imamo x = 2. Zato sada možemo pretpostaviti da su x, y celibrojevi različiti od 0.Diskutujmo sada znak brojeva x, y. Ako je x > 0, a y < 0, tada imamox 3 = y 3 + 2xy + 8 < 8, pa mora biti x = 1 i y 3 + 2y + 7 = 0, što ne zadovoljavanijedan ceo broj y. Ako je x < 0 i y > 0, tada je, s jedne strane, y 3 − x 3 =−2xy − 8 < −2xy, a s druge strane:y 3 − x 3 = y 3 + (−x) 3 y 2 + (−x) 2 −2xy,što je tako - de nemoguće.Dakle, x, y su istog znaka, xy > 0. Tada je0 < 2xy + 8 = x 3 − y 3 = (x − y)[(x − y) 2 + 3xy],pa kako je faktor u uglastoj zagradi pozitivan, to mora biti x > y. Sada ćemorazmotriti moguće vrednosti za x − y. Ukoliko bi bilo x − y 2, tada bismo imali2xy + 8 2(4 + 3xy) = 6xy + 8,što je nemoguće za xy > 0. Jedina mogućnost je, zbog toga, x − y = 1, odakle je2xy + 8 = 3xy + 1, tj. x(x − 1) = 7, što ne važi ni za jedno x ∈ Z.Stoga su na početku uočena trivijalna rešenja (0, −2) i (2, 0) ujedno i jedina.101


65. MMT, Mersch, Luksemburg, 1980. (Holandija)Naći sva celobrojna rešenja jednačinex 3 + x 2 y + xy 2 + y 3 = 8(x 2 + xy + y 2 + 1).Rešenje. Faktorizacijom leve strane i grupisanjem na desnoj strani, dobijamo:(x + y)(x 2 + y 2 ) = 4(x 2 + y 2 ) + 4(x + y) 2 + 8. (28)Pošto x = y = 0 nije rešenje posmatrane jednačine, možemo pisati(x + y)2x + y = 4 + 4x 2 + y 2 + 8x 2 + y 2 , (29)odakle sledi da je x + y > 4, tj. x + y 5. S druge strane, nejednakost kvadratnei aritmetičke sredine daje x 2 + y 2 1 2 (|x| + |y|)2 1 2 (x + y)2 , pa je x 2 + y 2 252 > 8, kao i (x + y)20 x 2 + y 2 2.Uzimajući u obzir ove nejednakosti, iz (29) zaključujemo da je x + y < 13, tj.x+y 12. Osim toga, pošto su brojevi x+y i x 2 +y 2 iste parnosti, iz (28) odmahsledi da je x + y parno. Prema tome, x + y ∈ {6, 8, 10, 12}.Ako sada, radi kraćeg zapisa, označimo x + y = 2a i xy = b, razmatranajednačina postaje2a(4a 2 − 2b) = 8(4a 2 − b + 1).Odavde lako možemo izraziti b:b = 2a3 − 8a 2 − 2a − 2= 2a 2 − 4a − 8 − 18a − 2 .Stoga (a − 2) | 18, pa pošto je 3 a 6, imamo tri slučaja: a ∈ {3, 4, 5}. Ovislučajevi za vrednosti b redom daju −20, −1 i 16. Dakle, (x, y) se dobija kao parrešenja jedne od sledeće tri kvadratne jednačine:t 2 − 6t − 20 = 0, t 2 − 8t − 1 = 0, t 2 − 10t + 16 = 0.Diskriminante ovih jednačina su redom 116, 68 i 36, što znači da samo trećajednačina ima celobrojna rešenja. Ta rešenja su 2 i 8, pa preostaje samo da proverimoda su (2, 8) i (8, 2) zaista rešenja jednačine date u zadatku.102


66. Bugarska, 1979.Dokazati da jednačinanema rešenja u skupu celih brojeva.x 2 + 5 = y 3Rešenje. Pretpostavimo suprotno. Razmotrimo najpre parnost brojeva x, y. Ako bix bilo neparno, tada bismo imali x 2 ≡ 1(mod 4), odakle bi sledilo y 3 ≡ 2(mod 4),što je nemoguće (pošto je y tada paran, pa je y 3 ≡ 0(mod 4)). Dakle, x je paranbroj, pa važi y 3 ≡ 1(mod 4), tj. y ≡ 1(mod 4). Ako sada uvrstimo x = 2u iy = 4v + 1 u datu jednačinu, slediodnosno4u 2 + 5 = (4v + 1) 3 = 64v 3 + 48v 2 + 12v + 1,u 2 + 1 = v(16v 2 + 12v + 3).Očito, 16v 2 + 12v + 3 = 4 · [v(4v + 3)] + 3, pa ovaj broj mora imati prost faktorp oblika 4t + 3 (u suprotnom bi svi njegovi prosti faktori bili oblika 4t + 1, što biznačilo da posmatrani broj daje ostatak 1 pri deljenju sa 4, a ne 3). Taj prost brojdeli u 2 + 1, pa dobijamo kontradikciju na gotovo identičan način kao u zadatku br.56 (b): naime, iz u 2 ≡ −1(mod p) imamou p−1 = u 4t+2 = (u 2 ) 2t+1 ≡ (−1) 2t+1 = −1(mod p),što je zbog p > 2 u suprotnosti sa malom Fermaovom teoremom.Komentar. Jednačina data u zadatku je specijalan slučaj tzv. Bašeove jednačinex 2 + k = y 3 ,gde je k ∈ Z. Ovu jednačinu je prvi posmatrao francuski matematičar ClaudeGaspard Bachet de Méziriac (1581–1638), po kome je dobila ime. Opšta Bašeovajednačina je i danas predmet aktivnog izučavanja u teoriji brojeva.103


67. MMT, Mariehamn, Finska, 1980. (Ma - darska)Neka je n 2 prirodan broj. Dokazati da jednačinax n + 1 = y n+1nema rešenje x, y ∈ N za koje važi (x, n + 1) ≠ 1.Rešenje. Transformišimo jednačinu:Pri tome je, očigledno,x n = y n+1 − 1 = (y − 1)(y n + . . . + y + 1).y n + . . . + y + 1 ≡ n + 1(mod y − 1),odakle sledi da d = (y − 1, y n + . . . + y + 1) deli n + 1.Ako bi sada bilo d = 1, tada bi oba uzajamno prosta broja y−1 i y n +. . .+y+1bili n-ti stepeni celih brojeva. Me - dutim,y n < y n + . . . + y + 1 < (y + 1) n ,pa dobijamo kontradikciju. Dakle, d > 1.Ali, ako je p prost delitelj od d, tada p | x n , odakle p | x. Istovremeno,podsetimo se da p | (n + 1). Otuda sledi tvr - denje zadatka.104


68. S.S.S.R., 1990. (republička olimpijada)Naći sve prirodne brojeve x, y za koje važi:7 x − 3 · 2 y = 1.Rešenje. Data jednačina je ekvivalentna sa:2 y−1 = 7x − 17 − 1 = 7x−1 + 7 x−2 + . . . + 1.Otuda je x = 1, y = 1, jedno od njenih rešenja, pa pretpostavimo sada da je y 2.Tada na levoj strani gornje jednačine stoji paran broj, a na desnoj zbir x neparnihbrojeva, odakle je x parno. To znači da možemo faktorisati:odnosno2 y−1 = (7 + 1)(7 x−2 + 7 x−4 + . . . + 1),2 y−4 = 7 x−2 + 7 x−4 + . . . + 1,odakle sledi da mora biti y 4. Odmah uočavamo da je x = 2, y = 4 drugorešenje. Zato pretpostavimo da je y 5. Tada je zbir x 2neparnih brojeva jednakparnom broju, pa je x 2paran broj, tj. x je deljivo sa 4. Sada možemo pisati:2 y−4 = (7 2 + 1)(7 x−4 + 7 x−8 + . . . + 1),što je zbog 7 2 + 1 = 50 nemoguće. Dakle, {(1, 1), (2, 4)} predstavlja traženi skuprešenja.105


69. Predlog za MMO, 1991. (Hong Kong)Naći sve prirodne brojeve x, y, z za koje važi:3 x + 4 y = 5 z .Rešenje. Posmatrajmo najpre datu jednačinu po modulu 3. Tada sledi2 z ≡ 5 z ≡ 1(mod 3),pa z mora biti paran broj, z = 2t. Stoga, važi3 x = 5 2t − 4 y = (5 t − 2 y )(5 t + 2 y ),odakle dobijamo da je 5 t + 2 y stepen trojke, dok je 5 t − 2 y ili jednak 1, ili stepentrojke. Me - dutim, drugi od ovih slucajeva je očito nemoguć, pošto bi tada broj(5 t + 2 y ) + (5 t − 2 y ) = 2 · 5 tbio deljiv sa 3. Prema tome, 5 t −2 y = 1 i 5 t +2 y = 3 x . Posmatrajući ove jednačinepo modulu 3, imamo (−1) t − (−1) y ≡ 1(mod 3) i (−1) t + (−1) y ≡ 0(mod 3),što znači da je t neparan, dok je y paran. Pišimo y = 2v, v ∈ N.Dakle, imamo jednačine 5 t − 4 v = 1, 5 t + 4 v = 3 x . Iz prve jednačine sledi5 t = 4 v + 1, što odmah povlači da je v = 1, jer bi u suprotnom (v 2) bilo5 t ≡ 1(mod 8), što je nemoguće, budući da je t neparno (naime, 5 2s+1 = 5 · 25 s ≡5(mod 8)). Pošto je v = 1, to je i t = 1, pa iz druge od dve navedene jednačineimamo x = 2. Znači, x = y = z = 2 je jedino rešenje zadatka.106


70. Predlog za MMO, 1978. ((a): Velika Britanija; (b): Holandija)(a) Prirodni brojevi x, y su takvi da je brojceo i deli 1978. Dokazati da je x = y.x 2 + y 2x + y(b) Dokazati da na kružnici opisanoj oko kvadrata sa temenima (0, 0), (1978, 0),(1978, 1978), (0, 1978), nema celobrojnih tačaka, sem navedenih.Rešenje. (a) Očigledno, zadatak se sastoji u diskutovanju jednačinex 2 + y 2 = m(x + y), (30)gde m | 1978. Naš cilj je da pokažemo da za posmatrane vrednosti m, ona imajedinstveno rešenje x = y = m. Primetimo da se nakon množenja sa 4, prebacivanjačlanova na levu stranu i dodavanja 2m 2 , (30) može pisati u obliku(2x − m) 2 + (2y − m) 2 = 2m 2 .Pošto su transformacije bile ekvivalentne, zadatak se sastoji u tome da se pokažeda se broj oblika 2m 2 (za posmatrane vrednosti m) može prikazati kao zbir dvakvadrata na samo jedan način (naime, kao m 2 + m 2 ).Za m = 1, ovo tvr - denje je očigledno. U suprotnom, ako je m > 1, pretpostavimoda se 2m 2 može predstaviti kao zbir dva kvadrata na neki drugi načinsem m 2 + m 2 . Na primer, neka je 2m 2 = a 2 + b 2 , pri čemu je a < b. Tada jea < m, pa s obzirom na to da m | 1978 = 2 · 23 · 43 (zbog čega je m proizvodrazličitih prostih brojeva), postoji prost faktor p od m koji ne deli a.Me - dutim, p | (a 2 + b 2 ). Po izboru p, važi (a, p) = 1, pa postoji prirodan broju tako da je au ≡ 1(mod p). S druge strane, p | ((au) 2 + (bu) 2 ), odakle je(bu) 2 ≡ −(au) 2 ≡ −1(mod p).Sada rezonujemo analogno kao i u <strong>zadaci</strong>ma br. 56 (b) i 66: stepenujući gornjukongruenciju sa p−12(pod pretpostavkom da je p > 2) i koristeći malu Fermaovuteoremu, sledi1 ≡ (bu) p−1 = ((bu) 2 ) p−12 ≡ (−1) p−12 (mod p),odakle je p ≡ 1(mod 4). Ali, 1978 = 2 · 23 · 43 nema takvih prostih faktora, paih samim tim ne može imati ni m. Prema tome, preostaje jedino mogućnost p = 2,107


dok za svaki drugi prost faktor q | m važi q | a i q | b. Skraćivanjem jednačinea 2 + b 2 = 2m 2 sa svim takvim prostim faktorima q, dobijamo jednačinu oblikac 2 + d 2 = 8. Jedino njeno rešenje je c = d = 2, tako da i u ovom slučaju slediželjeni zaključak a = b = m, koji okončava rešenje zadatka pod (a).(b) Jednačina date kružnice je(x − 989) 2 + (y − 989) 2 = 2 · 989 2 .Očito, ako je (x, y) celobrojna tačka na ovoj kružnici, onda je to i tačka(1978 − x, 1978 − y),koja je, primetimo, dijametralno suprotna tački (x, y). Me - dutim, kako polukružniluk zahvaćen tačkama (0, 1978) i (1978, 0) koji sadrži tačku (1978, 1978) ceo ležiu prvom kvadrantu, gornja jednačina ima rešenje u skupu celih brojeva ako i samoako ima rešenje u skupu celih brojeva 0.Me - dutim, 989 = 1978/2 = 23·43, što znači da razmatranja iz tačke (a) povlačeda je x − 989 = y − 989 = ±989, što daje upravo četiri tačke navedene u zadatku.Ove tačke čine dva dijametralno suprotna para, pa sledi da su to i jedine celobrojnetačke na posmatranoj kružnici, kao što se i tražilo.108


71. Predlog za MMO, 1978. (Francuska)(a) Neka za prirodne brojeve x, y, z važi xy − z 2 = 1. Dokazati da postoje nenegativniceli brojevi a, b, c, d tako da jex = a 2 + b 2 , y = c 2 + d 2 , z = ac + bd.(b) Dokazati: ako je p prost broj i p ≡ 1(mod 4), tada se p može predstaviti kaozbir dva kvadrata prirodnih brojeva.Rešenje. (a) Za ure - denu trojku prirodnih brojeva (x, y, z) za koju je xy − z 2 = 1kažemo da je loša ako je x y i ne postoje prirodni brojevi a, b, c, d tako da jex = a 2 + b 2 , y = c 2 + d 2 i z = ac + bd. U ovoj terminologiji, zadatak traži da sepokaže da loše trojke ne postoje.Pretpostavimo suprotno: neka je (x, y, z) neka loša trojka. Kako je xy = z 2 +1i x y, zaključujemo da važix 2 xy = z 2 + 1,odakle je x z (jer je z 2 + 1 < (z + 1) 2 ). Ako bi bilo x = z, imali bismo1 = xy − x 2 = x(y − x), što je moguće samo ukoliko je x = 1, y = 2. Me - dutim,tada imamo x = 0 2 + 1 2 , y = 1 2 + 1 2 i z = 0 · 1 + 1 · 1, što je kontradikcija sapretpostavkom da je posmatrana trojka loša. Prema tome, mora biti x < z.Neka je sada t = z − x, tj. z = x + t. Uvrštavajući ovo u polaznu jednačinu,imamo:1 = xy − (x + t) 2 = xy − x 2 − 2xt − t 2 = x(y − x − 2t) − t 2 .Dakle, trojka celih brojeva (x, y − x − 2t, t) = (x, x + y − 2z, z − x) je takode-rešenje jednačine xy − z 2 = 1. Pošto je ova jednačina simetrična po x, y, prvedve komponente svakog rešenja se mogu slobodno permutovati. Zbog toga, bezumanjenja opštosti možemo pretpostaviti da je x x + y − 2z (u suprotnomizvršimo transpoziciju prva dva broja u trojci). Osim toga, posmatrana trojka sesastoji od prirodnih brojeva. Naime, već smo videli da mora biti z − x > 0. Dalje,važi( ) x + y 2z 2 = xy − 1 < xy ,2pa je 2z < x + y, tj. x + y − 2z > 0.Sada pokazujemo da je ovako dobijena trojka tako - de loša. Pretpostavimo suprotno.Tada postoje prirodni brojevi m, n, p, q tako da važi:x = m 2 + n 2 , x + y − 2z = p 2 + q 2 , z − x = mp + nq.109


Iz ovog sistema dobijamoi zatim iz = m(m + p) + n(n + q),y = (m + p) 2 + (n + q) 2 ,kontradikcija sa pretpostavkom da je (x, y, z) bila loša trojka. Prema tome, i(x, x + y − 2z, z − x)je loša trojka.Me - dutim, važi z −x < z. Znači, ukoliko bismo pretpostavili da skup svih lošihtrojki nije prazan, sledio bi zaključak da me - du svim takvim trojkama (x, y, z) nepostoji trojka sa minimalnim z (jer se za svaku lošu trojku može konstruisati lošatrojka sa manjom trećom komponentom). To je, jasno, nemoguće, pa loših trojkizapravo i nema, tj. tvr - denje pod (a) je dokazano.(b) Neka je sada p = 4k + 1 prost broj. Po Vilsonovoj teoremi, tada važi(4k)! ≡ −1(mod p).Ali, važi i p−i ≡ −i(mod p) za 1 i 2k, pa množenjem svih ovih kongruencijasledi(4k)!(2k)! ≡ (−1)2k (2k)! (mod p),odakle je[(2k)!] 2 ≡ (4k)! ≡ −1(mod p).Ako označimo u = (2k)!, dobijamo da p | (u 2 + 1), odnosno da važipv − u 2 = 1za neki prirodan broj v. Primenom tvr - denja (a) na ovaj specijalan slučaj, dobijamotvr - denje (b).110


72. MMO, 1977. (SR Nemačka)Neka su a, b prirodni brojevi, i neka se pri deljenju a 2 + b 2 sa a + b dobija količnikq i ostatak r. Naći sve parove (a, b) za koje je q 2 + r = 1977.Rešenje. Brojevi a, b, q, r treba da zadovolje sledeće uslove:a 2 + b 2 = (a + b)q + r, 0 r < a + b, q 2 + r = 1977.Očito, q 2 1977, pa je q 44. Otuda jea 2 + b 2 < 44(a + b) + a + b = 45(a + b),što zajedno sa nejednakošću (a + b) 2 2(a 2 + b 2 ) dajea + b < 90,pa je r < 90. Odatle je, me - dutim, q 2 = 1977−r > 1887, tj. q > 43. Zaključujemoda je q = 44 i r = 41, pa preostaje da na - demo sva celobrojna rešenja jednačineNju možemo transformisati u oblika 2 + b 2 = 44(a + b) + 41.(a − 22) 2 + (b − 22) 2 = 1009.Odredimo sada celobrojna rešenja jedačine x 2 + y 2 = 1009, tako da je 0 x y.Kako mora biti y 2 1009 2y 2 , sledi 23 y 31. Otuda direktnom proveromdobijamo kao jedino rešenje y = 28, x = 15. Brojeve a, b odre - dujemo iz uslovapa je traženi skup rešenja{|a − 22|, |b − 22|} = {15, 28},{(7, 50), (37, 50), (50, 7), (50, 37)}.111


73. Velika Britanija, 1991.Dokazati: ako su a i b prirodni brojevi i a 2 + b 2 − a je deljivo sa 2ab, tada je apotpun kvadrat.Rešenje. Uslov zadatka možemo zapisati u vidu jednačinea 2 + b 2 − a = 2qab,sa parametrom q ∈ N. Neka je d = (a, b), a = rd, b = sd. Nakon uvrštavanja ugornju jednačinu i skraćivanja sa d, dobijamor 2 d + s 2 d − r = 2qrsd,što znači da d | r. Ali, s druge strane r | s 2 d, pa zbog (r, s) = 1 sledi r | d. Dakle,r = d i a = r 2 .Komentar. Kao uopštenje ovog zadatka, možemo postaviti problem nalaženja svihparova prirodnih brojeva (a, b) tako da za dato q ∈ N važia 2 + b 2 − a2ab= q.Kao što smo videli u gornjem rešenju, tada je a = r 2 , b = rs, gde je (r, s) rešenjejednačinex 2 − 2qxy + y 2 = 1. (31)Pošto će se jednačine veoma sličnog tipa pojaviti i u sledećim <strong>zadaci</strong>ma, u narednomćemo izložiti jedno opšte teorijsko razmatranje na osnovu kojeg ćemo rešitigornju jednačinu, a koje ćemo i kasnije koristiti.∗ ∗ ∗Razmatraćemo kvadratne diofantske jednačine oblikax 2 − mxy + y 2 = n, (32)gde su m, n celi brojevi, m > 0, pri čemu jednačinu rešavamo u N. Jedna odtemeljnih ideja koja se pojavljuje u analizi diofantskih jednačina jeste invarijantnostnjihovih skupova rešenja u odnosu na odre - dene transformacije u R 2 (tj. u Z 2 ). Neulazeći dublje u teoriju kvadratnih formi, nije redak slučaj da su skupovi rešenjadiofantskih jednačina invarijantni na odre - dene linearne transformacije, tako da iz112


pretpostavke da je (x, y) rešenje sledi da je rešenje i (ax+by, cx+dy) za pogodnoodabrane koeficijente a, b, c, d.Pošto želimo da primenimo (Fermaov) metod beskonačnog silaska, posmatraćemopreslikavanja kod kojih je a = 0 i b = 1, tj. kada je u pitanju transformacija(x, y) ↦→ (y, cx + dy). Naime, ako je x 2 − mxy + y 2 = n, tada jey 2 − my(cx + dy) + (cx + dy) 2 == n + (c 2 − 1)x 2 + (m − cm + 2cd)xy + (d 2 − dm)y 2 .Očigledno, uslovi c 2 − 1 = m − cm + 2cd = d 2 − dm = 0 daju koeficijente c, dsa željenim osobinama. Dakle, c = ±1; pri tome, c = 1 daje d = 0, i taj slučajnam nije interesantan, pošto daje transpoziciju para (x, y). Stoga biramo c = −1,odakle lako dobijamo d = m. Tako je skup celobrojnih rešenja jednačine (32)invarijantan na transformacijug : (x, y) ↦→ (y, my − x).Imajući u vidu simetričnost jednačine (32), možemo se ograničiti samo narešenja kod kojih je x > y > 0 (moguća su i rešenja kod kojih je x = y, alinsamo ukoliko je m ≠ 2 i2−mje potpun kvadrat, ili je m = 2 i n = 0). Zbogtoga, od značaja je da ustanovimo kada će rešenje oblika g(x, y) imati prvu komponentuveću od druge. Očito, ovaj uslov je izražen nejednakošću y > my − x, tj.(m − 1)y < x. S druge strane, komponente rešenja g(x, y) treba da budu prirodnibrojevi, pa zato imamo nejednakost my−x > 0, odnosno x < my. Dakle, možemorezimirati na sledeći način: ako je (x, y) prirodno rešenje jednačine (32) kod kojegje x > y i ako pri tome važe nejednakosti(m − 1)y < x < my, (33)tada je i g(x, y) = (x ′ , y ′ ) prirodno rešenje od (32) kod kojeg je x ′ > y ′ . Pritome, imamo x ′ < x. Na ovaj način, definisan je jedan iterativni postupak zadobijanje novih rešenja jednačine (32). On će se zaustaviti nakon konačno mnogokoraka, kada dobijemo ”minimalno” rešenje (x 0 , y 0 ) kod kojeg ne važi jedna odnejednakosti (33), tj. kod kojeg je ili x 0 (m − 1)y 0 , ili x 0 my 0 .Sada razlikujemo dva slučaja. Ako je n 0, tada za proizvoljno rešenje (x, y)od (32) imamox(my − x) = mxy − x 2 = y 2 − n y 2 < xy,tj. (m − 1)y < x. Tako je minimalnost rešenja u ovom slučaju ekvivalentna sax 0 my 0 . Pri tome, važi0 x 0 (my 0 − x 0 ) = y 2 0 − n,113


odakle je y 0 √ n. To zapravo znači da minimalnih rešenja ima konačno mnogo,jer za fiksirano y, (32) postaje kvadratna jednačina po x, zbog čega za sve y 0 ∈{1, . . . , ⌊ √ n⌋} imamo najviše dva ”kandidata” za x 0 tako da je (x 0 , y 0 ) minimalnorešenje. Stoga se minimalna rešenja mogu odrediti (u odsustvu boljeg metoda)direktnom proverom.S druge strane, ako je n < 0, tada (32) povlačix(my − x) = y 2 − n > 0.Dakle, svako rešenje (32) zadovoljava x < my, pa je minimalnost rešenja ekvivalentnasa x 0 (m − 1)y 0 (pri čemu odmah možemo pretpostaviti da je m > 2,jer za m = 1 imamo x 2 − xy + y 2 = ( x + y 22)+34 y2 0, dok za m = 2 važix 2 −2xy +y 2 = (x−y) 2 0). Nejednakost x 0 (m−1)y 0 je dalje ekvivalentnasax 0 (my 0 − x 0 ) x 0 y 0 ,pa iz x 0 (my 0 − x 0 ) = y0 2 + |n| dobijamo y 0(x 0 − y 0 ) |n|. Odavde je ili x 0 = y 0n(u slučaju da je2−m potpun kvadrat), ili y 0 |n|. Poslednja nejednakost, analognokao i u slučaju n 0, garantuje da imamo konačno mnogo minimalnih rešenja ipruža mogućnost da se ona odrede neposrednom proverom.Prema tome, ukoliko su nam poznata sva minimalna rešenja (x 0 , y 0 ), tada su(uz pretpostavku x > y) sva druga rešenja jednačine (32) oblika f k (x 0 , y 0 ), k ∈ N,gde je f inverzno preslikavanje od g,f : (x, y) ↦→ (mx − y, x),budući da za svako rešenje (x, y), x > y, g k (x, y) mora biti minimalno rešenje zadovoljno veliko k.∗ ∗ ∗U jednačini (31) koju razmatramo, parametri su m = 2q i n = 1. Na osnovumalopredašnjih - opštih zaključaka, sledi da su sva rešenja te jednačine oblikaf k (x 0 , y 0 ), k ∈ N 0 , gde je (x 0 , y 0 ) neko minimalno rešenje, a f(x, y) =(2qx − y, x). Budući da je n 0, minimalna rešenja su odredena - uslovomx 0 2qy 0 , odakle je y 0 1. Znači, y 0 = 1 i x 2 0 − 2qx 0 = 0 ⇒ x 0 = 2qpredstavlja jedino minimalno rešenje, pa su stoga sva rešenja jednačine (31) kodkojih je x > y oblika f k (2q, 1), k 0. Drugim rečima, sva tražena rešenja suiscrpljena parovima uzastopnih članova niza a k , k ∈ N, gde jeza k 1 i a 1 = 1, a 2 = 2q.a k+2 = 2qa k+1 − a k114


74. Vijetnam, 1992.Naći sva rešenja jednačineu skupu prirodnih brojeva.x 2 − 5xy + y 2 + 5 = 0Rešenje. U ovom zadatku je zapravo data jednačina (32) iz komentara prethodnogzadatka u specijalnom slučaju m = 5, n = −5. To odmah znači da ona nemarešenje za koje je x = y, pa se možemo ograničiti na rešenja kod kojih je x > y.Kako je n < 0, na osnovu razmatranja iz komentara prethodnog zadatka imamoda je minimalnost rešenja (x 0 , y 0 ) ekvivalentna sa x 0 4y 0 , što impliciray 0 (x 0 −y 0 ) 5 i y 0 5. Slučaj y 0 = 1 daje kvadratnu jednačinu x 2 0 −5x 0+6 = 0,odnosno minimalna rešenja (2, 1) i (3, 1). S druge strane, ako je y 0 2, tada jex 0 3, pa dobijamo(x 0 − y 0 ) 2 + 5 = 3x 0 y 0 18,odakle sledi x 0 − y 0 4 i y 0 (x 0 − y 0 ) 8, što je nemoguće. Znači, prethodna dvaminimalna rešenja su i jedina, zbog čega je skup svih rešenja razmatrane jednačine(uz ograničenje x > y):gde je f(x, y) = (5x − y, x).{f k (2, 1), f k (3, 1) : k ∈ N 0 },115


75. MMO, 1988. (SR Nemačka)Dokazati: ako je za neke prirodne brojeve a, b brojceo, tada je on potpun kvadrat.a 2 + b 2ab + 1Rešenje. Pretpostavimo da jea 2 + b 2ab + 1 = k.Tada slučaj a = b povlačik =2a2a 2 + 1 < 2,tj. k = 1, što jeste potpun kvadrat. Pretpostavimo zato, bez umanjenja opštosti, daje a > b. Tada je par (a, b) jedno rešenje jednačinex 2 − kxy + y 2 = ku skupu prirodnih brojeva. Me - dutim (prema razmatranjima iz komentara zadatkabr. 73), tada su rešenja ove jednačine i g r (a, b) za r = 1, 2, . . . , r 0 , gde jeg(x, y) = (y, ky − x),a g r 0(a, b) = (a 0 , b 0 ) je neko minimalno rešenje. Pri tome, imamo a 0 kb 0 (tj.a 0 −kb 0 0), jer je k 0. Naš cilj je da pokažemo da u ovoj nejednakosti mora davaži jednakost. Zbog toga, transformišemo datu jednačinu tako da se u njoj pojaviizraz a 0 − kb 0 :k = a 2 0 − ka 0 b 0 + b 2 0 = (a 0 − kb 0 ) 2 + ka 0 b 0 − k 2 b 2 0 + b 2 0 >> ka 0 b 0 − k 2 b 2 0 = kb 0 (a 0 − kb 0 ).Odavde je b 0 (a 0 − kb 0 ) < 1, odnosno a 0 − kb 0 0. Stoga važi a 0 = kb 0 , pauvrštavanjem u razmatranu jednačinu sledi k = b 2 0 . Dakle, k je ponovo potpunkvadrat, što je i trebalo dokazati.Komentar. Za k = 1, jedino rešenje je a = b = 1. Ako je k = l 2 za nekoprirodno l > 1, tada se iz gornjeg rešenja vidi da je jedino minimalno rešenje datosa b 0 = √ k = l i a 0 = kb 0 = l 3 . Prema tome, za posmatranu vrednost k, paroviprirodnih brojeva (a, b), a b, koji zadovoljavaju uslov zadatka jesu elementiskupa{f r (l 3 , l) : r ∈ N 0 },116


gde je f(x, y) = (l 2 x−y, x), tj. uzastopni članovi niza a s , s ∈ N, datog sa a 1 = l,a 2 = l 3 ia s+2 = l 2 a s+1 − a sza s 1.117


76. MMO, 1982. (Velika Britanija)(a) Neka je n ∈ N. Ako jednačinax 3 − 3xy 2 + y 3 = nima rešenje u skupu prirodnih brojeva, tada ona ima bar tri različita rešenja uskupu prirodnih brojeva. Dokazati.(b) Dokazati da gornja jednačina za n = 2891 nema rešenja u skupu prirodnihbrojeva.Rešenje. (a) Pošto se drugi i treći član leve strane date jednačine pojavljuju urazvoju kuba razlike (y − x) 3 = y 3 − 3xy 2 + 3x 2 y − x 3 , možemo transformisatix 3 − 3xy 2 + y 3 = (y − x) 3 − 3x 2 y + 2x 3 .Desna strana gornjeg identiteta po formi prilično podseća na levu, što motiviše daljetransformacije:(y − x) 3 − 3x 2 y + 2x 3 = (y − x) 3 − 3x 2 y + 3x 3 − x 3 == (y − x) 3 − 3(y − x)x 2 − x 3 = (y − x) 3 − 3(y − x)(−x) 2 + (−x) 3 .Dakle, ako levu stranu polazne jednačine označimo sa f(x, y), dobili smo da važif(x, y) = f(y − x, −x), što znači da je za svako rešenje (x, y) date jednačine par(y − x, −x) tako - de rešenje. Ponovo primenjujući ovo razmatranje, dobijamo da jei (−x − (y − x), −(y − x)) = (−y, x − y) rešenje (ako bismo još jednom primeniliposmatranu transformaciju, vratili bismo se na (x, y)). Prva dva rešenja su jednakaako i samo ako je x = y = 0, što je nemoguće zbog n 1. Slične kontradikcijedobijamo i iz pretpostavki o jednakosti bilo koja dva od tri navedena rešenja.(b) Pretpostavimo da za neke cele brojeve x, y važix 3 − 3xy 2 + y 3 = 2891 = 9 · 321 + 2.Tada je x 3 + y 3 ≡ −1(mod 3), pa imamo tri mogućnosti (kongruencije su pomodulu 3): x ≡ 0, y ≡ −1; x ≡ y ≡ 1; x ≡ −1, y ≡ 0. U prvom slučaju jex = 3t, y = 3u − 1 i2891 = x 3 − 3xy 2 + y 3 == 27t 3 − 9t(3u − 1) 2 + 27u 3 − 27u 2 + 9u − 1 ≡ −1(mod 9),kontradikcija. Analogno postupamo i u trećem slučaju, dok u drugom slučajuumesto rešenja (x, y) posmatramo pridruženo rešenje (y − x, −x), pa imamo prviod navedena tri slučaja, za koji smo već pokazali da je nemoguć.118


77. Predlog za MMO, 1988. (S.S.S.R.)Neka su a, b, c celi brojevi različiti od 0. Poznato je da jednačinaax 2 + by 2 + cz 2 = 0ima celobrojno rešenje različito od x = y = z = 0. Dokazati da jednačinaima racionalno rešenje.ax 2 + by 2 + cz 2 = 1Rešenje. Neka su x 0 , y 0 , z 0 celi brojevi takvi da jeax 2 0 + by 2 0 + cz 2 0 = 0, (34)pri čemu je bar jedan od njih ≠ 0. Primetimo da tada najviše jedan od ovih brojevamože biti jednak 0. Bez umanjenja opštosti, neka je x 0 , z 0 ≠ 0.Naš cilj je da odredimo racionalne brojeve x, y, z tako da važiax 2 + by 2 + cz 2 = 1, (35)Najpre ćemo pomnožiti (34) sa ( z z 0) 2 kako bismo eliminisali bar jedan od nepoznatihparametara a, b, c (u ovom slučaju c). Ako radi kraćeg zapisa označimo t =zz 0, što znači da imamo smenu z = z 0 t, dobijamoOduzimajući ovo od (35), imamoax 2 0t 2 + by 2 0t 2 + cz 2 = 0.a(x 2 − x 2 0t 2 ) + b(y 2 − y 2 0t 2 ) = 1.Ako sada uspemo da prona - demo bar jedno racionalno rešenje (x, y, t) ove jednačine,i polazni problem će biti rešen. Možemo pokušati da potražimo rešenje za kojeje a(x 2 − x 2 0 t2 ) = 1 i b(y 2 − y 2 0 t2 ) = 0. Drugi uslov će biti ispunjen ako je npr.y = y 0 t. S druge strane, prva jednačina se može pisati kaoa(x − x 0 t)(x + x 0 t) = 1. (36)Medutim, - ako su sada r, s proizvoljni racionalni brojevi takvi da je rs = 1 a, tada ćerešenje sistema linearnih jednačinax − x 0 t = r,x + x 0 t = s,119


iti istovremeno i rešenje jednačine (36). Ovaj sistem ima determinantu 2x 0 ≠ 0 iracionalne koeficijente, pa za njegovo rešenje (x, t) mora važiti x, t ∈ Q. Tada su iy = y 0 t i z = z 0 t tako - de racionalni brojevi. Iz prethodnih razmatranja se lako vidida ovako odre - deni brojevi x, y, z zaista predstavljaju rešenje od (35), pa je zadatakrešen.Komentar. Na primer, ako u gornjem rešenju uzmemo r = 1, s = 1 a , dobićemoiodakle jeix = 1 + a2at = 1 − a2ax 0,y = (1 − a)y 02ax 0z = (1 − a)z 02ax 0.120


78. Predlog za MMO, 1989. (Južna Koreja)Neka su a, b celi brojevi koji nisu potpuni kvadrati. Dokazati: ako jednačinax 2 − ay 2 − bz 2 + abw 2 = 0ima netrivijalno celobrojno rešenje, tada to važi i za jednačinux 2 − ay 2 − bz 2 = 0.Rešenje. Najpre, a, b ≠ 0, jer a, b nisu potpuni kvadrati. Tako - de, a, b ne mogubiti istovremeno negativni, pa pretpostavimo, bez umanjenja opštosti, da je a > 0.Neka je (X, Y, Z, W ) netrivijalno rešenje jednačinex 2 − ay 2 − bz 2 + abw 2 = 0.Ideja se sastoji u tome da, slično kao i u nizu prethodnih zadataka, od posmatranogrešenja dobijemo (pogodnom transformacijom) celobrojno rešenje za x 2 − ay 2 −bz 2 = 0.Naravno, dve uočene jednačine se razlikuju u broju članova sa leve strane: prvaima četiri, a druga tri člana. To sugeriše da izvršimo grupisanje:X 2 − aY 2 − b(Z 2 − aW 2 ) = 0.”Nedostatak” je sada u tome da izraz u zagradi nije, u opštem slučaju, potpunkvadrat, što se, me - dutim, može ”popraviti” množenjem gornje jednakosti sa Z 2 −aW 2 . Tako imamoS druge strane, važi:(X 2 − aY 2 )(Z 2 − aW 2 ) − b(Z 2 − aW 2 ) 2 = 0.(X 2 − aY 2 )(Z 2 − aW 2 ) = X 2 Z 2 − aY 2 Z 2 − aX 2 W 2 + a 2 Y 2 W 2 == (X 2 Z 2 + 2aXY ZW + a 2 Y 2 W 2 ) − a(Y 2 Z 2 + 2XY ZW + X 2 W 2 ) =Zbog toga, ako označimo= (XZ + aY W ) 2 − a(Y Z + XW ) 2 .x 0 = XZ + aY W,y 0 = Y Z + XW,z 0 = Z 2 − aW 2 ,121


dobijamox 2 0 − ay 2 0 − bz 2 0 = 0.Očigledno, x 0 , y 0 , z 0 su celi brojevi. Preostaje da pokažemo da je bar jedan odnjih ≠ 0. Tačnije, tvrdimo da je z 0 ≠ 0. U suprotnom, sledilo bi Z 2 = aW 2 .Pošto a nije potpun kvadrat, zaključili bismo da je Z = W = 0. Ali, tada bismo izpolazne jednačine dobili X 2 −aY 2 = 0, tj. X 2 = aY 2 , što povlači X = Y = 0, štoje kontradikcija sa pretpostavljenom netrivijalnošću rešenja (X, Y, Z, W ). Dakle,(x 0 , y 0 , z 0 ) je netrivijalno celobrojno rešenje jednačine x 2 − ay 2 − bz 2 = 0.122


79. Predlog za MMO, 1988. (Grčka)Naći celobrojno rešenje jednačinex 2 1 + x 2 2 + . . . + x 2 29 = 29x 1 x 2 . . . x 29tako da za bar jedno 1 k 29 važi x k 1988 2 .Rešenje. Kao i u nekoliko prethodnih zadataka, ideja rešenja se sastoji u tome dase polazeći od nekog rešenja date jednačine konstruiše novo. Uz pogodnu transformaciju,komponente rešenja će biti sve veće, pa ćemo, polazeći od očiglednogrešenja x 1 = . . . = x 29 = 1, nakon odgovarajućeg broja iteracija dobiti ”dovoljnoveliko” rešenje.Naravno, voleli bismo da ta transformacija bude što je moguće jednostavnija.Naime, pokušaćemo da izmenimo samo jednu komponentu rešenja, odnosno daod (x 1 , x 2 , . . . , x 29 ) pre - demo na (y, x 2 , . . . , x 29 ). Kako bi ova druga 29-orka bilarešenje, potrebno je i dovoljno da budey 2 + x 2 2 + . . . + x 2 29 = 29yx 2 . . . x 29 .Oduzimajući od ovoga polaznu jednačinu, dobijamoy 2 − x 2 1 = 29yx 2 . . . x 29 − 29x 1 x 2 . . . x 29 = 29(y − x 1 )x 2 . . . x 29 ,Nakon skraćivanja sa y − x 1 (što ima smisla, pošto očigledno želimo da bude y ≠x 1 ), imamo y + x 1 = 29x 2 . . . x 29 . Lako se proverava day = 29x 2 . . . x 29 − x 1zaista daje novo rešenje razmatrane jednačine.Sada treba da obezbedimo da uočena transformacija zaista povećava komponenterešenja. Ako pretpostavimo da smo na početku imalitada jex 1 x 2 . . . x 29 ,29x 2 . . . x 29 − x 1 29x 29 − x 29 = 28x 29 > x 29 , (37)pa je, imajući u vidu da je razmatrana jednačina simetrična po svim promenljivama,svrsishodno da posmatramo našu transformaciju rešenja u obliku(x 1 , x 2 , . . . , x 29 ) → (x 2 , . . . , x 29 , 29x 2 . . . x 29 − x 1 ),123


kako bismo očuvali neopadajući poredak i time obezbedili da nejednakost (37) važinakon svake primene gornje transformacije.Prema tome, ako po - demo od već uočenog trivijalnog rešenja (1, 1, . . . , 1), tadaredom dobijamo sledeća rešenja:(1, 1, . . . , 1, 28),(1, 1, . . . , 28, 29 · 28 − 1),(1, 1, . . . , 28, 29 · 28 − 1, 29 · 28(29 · 28 − 1) − 1),(1, 1, . . . , 29 · 28(29 · 28 − 1)(29 · 28(29 · 28 − 1) − 1) − 1).Me - dutim, poslednja komponenta poslednjeg rešenja je očito veća od 1988 2 .124


80. Kina, 1991.Rešiti jednačinu u skupu prirodnih brojeva:x 2n+1 − y 2n+1 = xyz + 2 2n+1 ,pri čemu važe ograničenja n 2 i z 5 · 2 2n .Rešenje. Iz date jednačine je očigledno da važi x > y 1. Tako - de, primetimoda x i y moraju biti iste parnosti. Zaista, ako je y parno, tada su svi članovi datejednačine osim x 2n+1 parni, odakle x mora biti parno. Slično, parnost x povlačiparnost y. Zbog toga je x − y 2 i x 3.S obzirom da imamo rastavljanjevaže nejednakostix 2n+1 − y 2n+1 = (x − y)(x 2n + x 2n−1 y + . . . + y 2n ),x 2n+1 − y 2n+1 > (x − y)(x 2n + x 2n−1 y) > (x − y) · 2x 2n−1 y 4x 2n−1 y.S druge strane, iz z 5 · 2 2n sledix 2n+1 − y 2n+1 5 · 2 2n xy + 2 2n+1 = 2 2n (5xy + 2).Kombinujući ove dve nejednakosti, dobijamo4x 2n−1 y < 2 2n (5xy + 2),odnosno, nakon deljenja sa 2 2n xy,( x) 2n−2 2 < 5 +2xy < 6.(Druga nejednakost sledi iz xy > 2.) Kako je 2n − 2 2, zaključujemo da jex < 2 √ 6 < 5, pa su jedine mogućnostiPrva od njih povlači nejednakost(x, y) ∈ {(3, 1), (4, 2)}.( 32) 2n−2< 6,odakle je n 3, dok druga daje2 2n−2 < 6,125


što je moguće samo ako je n = 2.Za x = 3, y = 1, n = 2 dobijamo z = 70, za iste vrednosti x, y i n = 3imamo z = 686, dok za x = 4, y = 2 sledi z = 120. Pošto za n = 3 ograničenjedato u zadatku glasi z 320, odbacujemo drugo rešenje, a kako za n = 2 morabiti z 80, odbacujemo i treće rešenje. Prema tome, jedino rešenje sa traženimsvojstvima je(x, y, z) = (3, 1, 70)i n = 2.126


81. Predlog za MMO, 1995. (Bugarska)Naći sve prirodne brojeve x, y za koje važigde je z = (x, y).x + y 2 + z 3 = xyz,Rešenje. Neka je x = az i y = bz, pri čemu je (a, b) = 1. Data jednačina sadapoprima oblik:a + b 2 z + z 2 = abz 2 .Otuda je a = cz za neki prirodan broj c, pa imamo jednačinuc + b 2 + z = cbz 2 ,odakle neposredno nalazimo (pošto je bz 2 ≠ 1, u suprotnom bi bilo y = b = z = 1,tj. x + 2 = x)c = b2 + zbz 2 − 1 . (38)Množenjem sa z 2 (koje je očigledno pogodno iz ”tehničkih” razloga) dobijamoz 2 c = b2 z 2 + z 3bz 2 − 1= b + b + z3bz 2 − 1 . (39)Drugi sabirak u gornjoj jednačini mora biti prirodan broj, pa je on 1. Toje ekvivalentno nejednakosti b + z 3 bz 2 − 1, tj. b(z 2 − 1) z 3 + 1. Nakonskraćivanja sa z + 1, slediAko je z = 1, tada iz (39) imamob(z − 1) z 2 − z + 1.c = b2 + 1b − 1 = b + 1 + 2b − 1 ,pa je b = 2 ili b = 3, a odgovarajuća rešenja su (x, y) ∈ {(5, 2), (5, 3)}. Usuprotnom, dobijamoZa z = 2, jednačina (39) postajeb z2 − z + 1z − 1= z + 1z − 1 .16c = 16b2 + 324b − 1= 4b + 1 + 334b − 1 ,127


što daje rešenja b = 1 i b = 3, tj. (x, y) ∈ {(4, 2), (4, 6)}. S druge strane, za z 31važiz−1< 1, pa zaključujemo b < z + 1, tj. b z. Uvrštavajući ovu nejednakostu (38) i koristeći b 1, sledic < z2 + zz 2 − 1 =zz − 1 = 1 + 1z − 1 < 2.Znači, c = 1. Prema tome, b je rešenje kvadratne jednačineb 2 − z 2 b + (z + 1) = 0,što povlači da je diskriminanta te jednačine z 4 −4z −4 potpun kvadrat celog broja.Ali, s obzirom na z 3, važe stroge nejednakosti(z 2 − 1) 2 < z 4 − 4z − 4 < (z 2 ) 2 ,i time smo dobili kontradikciju. Dakle, skup rešenja je{(4, 2), (4, 6), (5, 2), (5, 3)}.128


82. Predlog za MMO, 1988. (Vijetnam); Jugoslavija, 1990.Neka je n prirodan broj, A skup koji se sastoji od tačno n + 1 prirodnih brojeva,a P skup svih prostih faktora elemenata skupa A. Ako je |P | n, dokazati dapostoji B ⊆ A, B ≠ ∅, tako da je proizvod elemenata skupa B potpun kvadrat.Rešenje. Neka je A = {a 1 , a 2 , . . . , a n+1 }. Po uslovima zadatka, postoji n različitihprostih brojeva p 1 , . . . , p n tako da za sve 1 i n + 1 važia i = p k i11 . . . p k innza neke nenegativne cele brojeve k ij , 1 j n. Prema tome, svaki proizvodπ(B) elemenata nepraznog podskupa B ⊆ A se takode - može napisati u oblikuπ(B) = p β 11 . . . pβ nn za odgovarajuće β 1 , . . . , β n 0.Jasno, π(B) će biti potpun kvadrat ako i samo ako su svi brojevi β j , 1 j n,parni. Zbog toga, svakom broju oblika l = p α 11 . . . pα nn pridružujemo niz elemenataskupa {0, 1},e(l) = (e 1 , . . . , e n ),tako da je e i = 0 ako je α i parno, a u suprotnom je e i = 1. Sada je l potpunkvadrat ako i samo ako se e(l) sastoji od samih nula. Osim toga, ako je e(l 1 ) =(e 1 , . . . , e n ) i e(l 2 ) = (e ′ 1 , . . . , e′ n), tada jegde za sve 1 j n važie(l 1 l 2 ) = (e ′′1, . . . , e ′′ n),e ′′j ≡ e i + e ′ i(mod 2). (40)Nepraznih podskupova od A ima 2 n+1 − 1. S druge strane, nizova dužine nkoji se sastoje od nula i jedinica ima 2 n . Kako je 2 n+1 − 1 > 2 n za sve n 1,po Dirihleovom principu postoje dva različita neprazna podskupa B, C ⊆ A takoda je e(π(B)) = e(π(C)). Zbog toga, iz (40) sledi da je e(π(B)π(C)) nula-niz,tj. da je π(B)π(C) potpun kvadrat. Me - dutim, pošto za svaka dva disjunktna skupaX, Y ⊆ A važiπ(X)π(Y ) = π(X ∪ Y )(po samoj definiciji oznake π), imamo da jeπ(B) = π(B \ C)π(B ∩ C) i π(C) = π(B ∩ C)π(C \ B)(pri čemu smo po potrebi dodefinisali da je π(∅) = 1). Otuda jeπ(B)π(C) = π(B \ C) · [π(B ∩ C)] 2 · π(C \ B),129


zbog čega jeπ(B \ C)π(C \ B) = π((B \ C) ∪ (C \ B))potpun kvadrat. Budući da je B ≠ C, skup (B \ C) ∪ (C \ B) je neprazan, čimesmo dobili podskup od A sa traženim osobinama.130


83. MMO, 1990. (Turska)Konstruisati funkciju f : Q + → Q + (gde Q + označava skup pozitivnih racionalnihbrojeva) tako da važif(xf(y)) = f(x)yza sve x, y ∈ Q + .Rešenje. Najpre, funkcija f je injektivna, jer iz pretpostavke f(y 1 ) = f(y 2 ) i datefunkcionalne jednačine očito sledi y 1 = y 2 (pošto je f(x) > 0). Ako sada stavimoy = 1, imamo f(xf(1)) = f(x) za sve x ∈ Q + , odakle je xf(1) = x i f(1) = 1.S druge strane, x = 1 dajef(f(y)) = 1 yza sve y ∈ Q + . (41)Primenjujući f na gornju jednakost još jednom, dobijamo f( 1 y ) = 1f(y) . Najzad,ako sada u polaznu jednačinu uvrstimo y = f( 1 z), sledif(xz) = f(x)f(z) za sve x, z ∈ Q + . (42)Obratno, veoma lako se proverava da svaka funkcija f : Q + → Q + koja zadovoljavauslove (41) i (42) zadovoljava i funkcionalnu jednačinu datu u zadatku.Imajući u vidu osnovnu teoremu aritmetike, svaki pozitivan racionalan brojmože se zapisati u obliku p α 11 . . . pα nn , gde su p 1 , . . . , p n prosti brojevi i α 1 , . . . ,α n ∈ Z. Zbog (42), mora da važif(p α 11 . . . pα nn ) = f(p 1 ) α 1. . . f(p n ) α n,pa je tražena funkcija u potpunosti odredena - svojim vrednostima na skupu prostihbrojeva. Drugim rečima, ako odredimo f(p) za sve proste brojeve p tako da (nakonproširivanja f na Q + putem gornje jednakosti) važi f(f(p)) = 1 p, tada se lakopokazuje da dobijena funkcija zadovoljava uslove (41) i (42). Preostaje da seprimeti da je željeni cilj postignut ako definišemof(p i ) ={pi+1 ako je i neparno,1p i−1ako je i parno,gde je p i , i ∈ N, niz prostih brojeva, pošto tada za neparno i važi f(f(p i )) =f(p i+1 ) = 1 p i, dok za parno i imamo f(f(p i )) = f(p −1i−1 ) = f(p i−1) −1 = 1 p i.131


Tabelarni pregled porekla zadatakaZemlja porekla MMO pMMO MMT(p) BMO(p) NOAustralija 21,45Belgija 22 15,39,64 34Bugarska 35,44,50,81 53,60 46,66Čehoslovačka 32 12,17,48Francuska 71Grčka 79Holandija 70b 65Hong Kong 69Irska 8,37Jugoslavija 38 47 82Južna Koreja 9,78Kanada 56Kina 80Kipar 55Kolumbija 49Kuba 52Madarska - 7,19 67 2a,18,24,30,31,42,62Novi Zeland 4Poljska 20 23Rumunija 29,59 2b,25,40,41, 46 10,27,28,57c57abS.A.D. 6,16,26,58,63SR Nemačka 5,72,75 3S.S.S.R. 36 13,33,51,77 1,61,68Švedska 43Turska 83V.Britanija 76 11,14,70a 73Vijetnam 54,82 74Legenda: MMO – zadatak na Me - dunarodnoj olimpijadi, pMMO – predlog za Me - dunarodnu olimpijadu,MMT(p) – <strong>zadaci</strong> na me - dunarodnim takmičenjima 1980. i predlozi, BMO(p) – <strong>zadaci</strong> na Balkanskimolimpijadama i predlozi, NO – nacionalne olimpijade132

Hooray! Your file is uploaded and ready to be published.

Saved successfully!

Ooh no, something went wrong!